Quiz-summary
0 of 30 questions completed
Questions:
- 1
- 2
- 3
- 4
- 5
- 6
- 7
- 8
- 9
- 10
- 11
- 12
- 13
- 14
- 15
- 16
- 17
- 18
- 19
- 20
- 21
- 22
- 23
- 24
- 25
- 26
- 27
- 28
- 29
- 30
Information
Premium Practice Questions
You have already completed the quiz before. Hence you can not start it again.
Quiz is loading...
You must sign in or sign up to start the quiz.
You have to finish following quiz, to start this quiz:
Results
0 of 30 questions answered correctly
Your time:
Time has elapsed
Categories
- Not categorized 0%
- 1
- 2
- 3
- 4
- 5
- 6
- 7
- 8
- 9
- 10
- 11
- 12
- 13
- 14
- 15
- 16
- 17
- 18
- 19
- 20
- 21
- 22
- 23
- 24
- 25
- 26
- 27
- 28
- 29
- 30
- Answered
- Review
-
Question 1 of 30
1. Question
A 68-year-old patient, Mr. Ebenezer Moreau, undergoing chemotherapy for acute myeloid leukemia, received a unit of packed red blood cells. Three days post-transfusion, Mr. Moreau develops a fever, chills, and hypotension. Blood cultures drawn from Mr. Moreau grow *Yersinia enterocolitica*. The blood bank investigation confirms that the implicated unit of red blood cells was collected during a period when the donor was asymptomatic but later developed *Yersinia* infection. According to federal regulations and established guidelines for transfusion-transmitted infections, which of the following actions is the MOST critical and legally mandated responsibility of the transfusion service in this scenario?
Correct
The correct answer is that the transfusion service must notify the FDA about the confirmed case of post-transfusion *Yersinia enterocolitica* sepsis. This is mandated by federal regulations and guidelines to ensure proper tracking and investigation of transfusion-transmitted infections. While quarantining and recalling the implicated blood product is a crucial step in preventing further harm, and implementing corrective actions within the blood bank is essential for preventing recurrence, the FDA notification is a legal requirement. Furthermore, while notifying the blood donor center is important for donor follow-up and future deferral considerations, it does not fulfill the regulatory obligation to inform the FDA, which oversees the safety and efficacy of the nation’s blood supply. The FDA uses this information to monitor trends, identify potential risks, and implement nationwide safety measures. Therefore, the notification to the FDA is paramount and must be performed promptly upon confirmation of the transfusion-transmitted bacterial infection. Blood banks are required to report suspected and confirmed transfusion-related fatalities to the FDA within specific timeframes, and severe bacterial infections like *Yersinia* sepsis fall under this reporting umbrella due to their potential for significant morbidity and mortality. Adherence to these reporting requirements is crucial for maintaining the integrity and safety of the blood supply.
Incorrect
The correct answer is that the transfusion service must notify the FDA about the confirmed case of post-transfusion *Yersinia enterocolitica* sepsis. This is mandated by federal regulations and guidelines to ensure proper tracking and investigation of transfusion-transmitted infections. While quarantining and recalling the implicated blood product is a crucial step in preventing further harm, and implementing corrective actions within the blood bank is essential for preventing recurrence, the FDA notification is a legal requirement. Furthermore, while notifying the blood donor center is important for donor follow-up and future deferral considerations, it does not fulfill the regulatory obligation to inform the FDA, which oversees the safety and efficacy of the nation’s blood supply. The FDA uses this information to monitor trends, identify potential risks, and implement nationwide safety measures. Therefore, the notification to the FDA is paramount and must be performed promptly upon confirmation of the transfusion-transmitted bacterial infection. Blood banks are required to report suspected and confirmed transfusion-related fatalities to the FDA within specific timeframes, and severe bacterial infections like *Yersinia* sepsis fall under this reporting umbrella due to their potential for significant morbidity and mortality. Adherence to these reporting requirements is crucial for maintaining the integrity and safety of the blood supply.
-
Question 2 of 30
2. Question
A 32-year-old male involved in a high-speed motor vehicle accident is admitted to the trauma center. He is actively bleeding from multiple sites and requires a massive transfusion protocol (MTP). Initial laboratory results, drawn after the administration of 6 units of packed red blood cells, show: PT/INR 1.8 (reference range: 0.9-1.1), aPTT 55 seconds (reference range: 25-35 seconds), fibrinogen 80 mg/dL (reference range: 200-400 mg/dL), and platelet count 75 x 10^9/L (reference range: 150-400 x 10^9/L). A thromboelastography (TEG) assay indicates decreased clot strength. Which of the following transfusion strategies is MOST appropriate at this time, considering the patient’s clinical condition and laboratory results, in accordance with best practices for massive transfusion and coagulation management?
Correct
The correct answer involves understanding the principles of massive transfusion protocols (MTPs) and their impact on coagulation testing, particularly in trauma patients. MTPs aim to rapidly replace blood volume and maintain oxygen-carrying capacity. However, they can dilute the patient’s coagulation factors and platelets, leading to dilutional coagulopathy. The PT/INR and aPTT are prolonged due to the reduced concentration of coagulation factors. Fibrinogen, being consumed during trauma and further diluted by transfused products lacking fibrinogen (like packed red blood cells), also decreases. Platelet counts are reduced due to both consumption and dilution. Thromboelastography (TEG) or Rotational Thromboelastometry (ROTEM) provides a more comprehensive assessment of coagulation, including clot strength and stability, which can guide targeted component therapy. In this scenario, the trauma patient is receiving a MTP, and the lab results reflect dilutional coagulopathy. The best course of action is to administer a combination of blood components designed to address the specific deficiencies identified, such as cryoprecipitate (to increase fibrinogen), platelets (to increase platelet count), and potentially prothrombin complex concentrate (PCC) to address factor deficiencies indicated by the prolonged PT/INR. While continuing the MTP is necessary for volume replacement, it must be balanced with coagulation support to prevent further bleeding complications.
Incorrect
The correct answer involves understanding the principles of massive transfusion protocols (MTPs) and their impact on coagulation testing, particularly in trauma patients. MTPs aim to rapidly replace blood volume and maintain oxygen-carrying capacity. However, they can dilute the patient’s coagulation factors and platelets, leading to dilutional coagulopathy. The PT/INR and aPTT are prolonged due to the reduced concentration of coagulation factors. Fibrinogen, being consumed during trauma and further diluted by transfused products lacking fibrinogen (like packed red blood cells), also decreases. Platelet counts are reduced due to both consumption and dilution. Thromboelastography (TEG) or Rotational Thromboelastometry (ROTEM) provides a more comprehensive assessment of coagulation, including clot strength and stability, which can guide targeted component therapy. In this scenario, the trauma patient is receiving a MTP, and the lab results reflect dilutional coagulopathy. The best course of action is to administer a combination of blood components designed to address the specific deficiencies identified, such as cryoprecipitate (to increase fibrinogen), platelets (to increase platelet count), and potentially prothrombin complex concentrate (PCC) to address factor deficiencies indicated by the prolonged PT/INR. While continuing the MTP is necessary for volume replacement, it must be balanced with coagulation support to prevent further bleeding complications.
-
Question 3 of 30
3. Question
Dr. Anya Sharma is consulting on a case involving a 62-year-old patient, Mr. Jian, undergoing chemotherapy for acute myeloid leukemia (AML). Mr. Jian, who weighs 75 kg and has a height of 175 cm, has developed severe thrombocytopenia with a current platelet count of 15,000/µL. The hematologist wants to increase Mr. Jian’s platelet count to at least 65,000/µL before his next chemotherapy cycle. The blood bank uses apheresis platelets with an average platelet count of \(3.0 \times 10^{11}\) per unit. Given that Mr. Jian’s body surface area (BSA) is 1.8 \(m^2\), and considering the formula:
\[
\text{Platelets needed} = \frac{\text{Desired increment} \times \text{BSA} \times 10^{11}}{\text{Platelet count per unit}}
\]How many apheresis platelet units are required to achieve the desired platelet increment of 50,000/µL for Mr. Jian?
Correct
To calculate the required number of platelets, we first need to determine the platelet increment needed, accounting for body surface area (BSA). The desired increment is 50,000/µL. The BSA is 1.8 \(m^2\). The formula to calculate the number of platelets needed is:
\[
\text{Platelets needed} = \frac{\text{Desired increment} \times \text{BSA} \times 10^{11}}{\text{Platelet count per unit}}
\]The platelet count per apheresis unit is \(3.0 \times 10^{11}\). Plugging in the values:
\[
\text{Platelets needed} = \frac{50,000/\mu L \times 1.8 \, m^2 \times 10^{11}}{3.0 \times 10^{11}} = \frac{5 \times 10^4 \times 1.8 \times 10^{11}}{3.0 \times 10^{11}}
\]\[
\text{Platelets needed} = \frac{9 \times 10^{15}}{3.0 \times 10^{11}} = 3
\]Therefore, 3 apheresis platelet units are needed to achieve the desired increment.
This calculation underscores the importance of considering patient-specific factors like BSA when determining transfusion dosages. Underestimation can lead to therapeutic failure, while overestimation exposes the patient to unnecessary risks. Furthermore, variations in platelet content among different apheresis units, as well as factors like platelet refractoriness, can impact the actual post-transfusion platelet count. Therefore, post-transfusion platelet counts are crucial for evaluating the effectiveness of the transfusion and adjusting subsequent dosages. Understanding these principles is essential for SBB-certified professionals to ensure optimal patient outcomes in transfusion medicine.
Incorrect
To calculate the required number of platelets, we first need to determine the platelet increment needed, accounting for body surface area (BSA). The desired increment is 50,000/µL. The BSA is 1.8 \(m^2\). The formula to calculate the number of platelets needed is:
\[
\text{Platelets needed} = \frac{\text{Desired increment} \times \text{BSA} \times 10^{11}}{\text{Platelet count per unit}}
\]The platelet count per apheresis unit is \(3.0 \times 10^{11}\). Plugging in the values:
\[
\text{Platelets needed} = \frac{50,000/\mu L \times 1.8 \, m^2 \times 10^{11}}{3.0 \times 10^{11}} = \frac{5 \times 10^4 \times 1.8 \times 10^{11}}{3.0 \times 10^{11}}
\]\[
\text{Platelets needed} = \frac{9 \times 10^{15}}{3.0 \times 10^{11}} = 3
\]Therefore, 3 apheresis platelet units are needed to achieve the desired increment.
This calculation underscores the importance of considering patient-specific factors like BSA when determining transfusion dosages. Underestimation can lead to therapeutic failure, while overestimation exposes the patient to unnecessary risks. Furthermore, variations in platelet content among different apheresis units, as well as factors like platelet refractoriness, can impact the actual post-transfusion platelet count. Therefore, post-transfusion platelet counts are crucial for evaluating the effectiveness of the transfusion and adjusting subsequent dosages. Understanding these principles is essential for SBB-certified professionals to ensure optimal patient outcomes in transfusion medicine.
-
Question 4 of 30
4. Question
Dr. Anya Sharma, a hematologist, is alerted by the nursing staff about a patient, Mr. Ben Carter, who has developed a fever, chills, and lower back pain approximately 30 minutes into a red blood cell transfusion. Mr. Carter has a history of multiple transfusions due to chronic anemia. The nurse immediately stopped the transfusion and notified Dr. Sharma. According to AABB standards and FDA regulations, what is the MOST appropriate immediate course of action that Dr. Sharma should direct the nursing staff to take? Consider the potential severity of transfusion reactions and the importance of a systematic approach to investigation and management.
Correct
The correct course of action involves prioritizing the immediate safety of the patient while adhering to regulatory requirements for transfusion reaction investigations. The initial step is to immediately stop the transfusion to prevent further exposure to the potentially harmful component. Maintaining the IV line with normal saline ensures venous access for potential emergency medication administration. A thorough clerical check of patient identification, blood product compatibility, and transfusion paperwork is crucial to identify any errors that may have led to the reaction. Notifying the transfusion service and the patient’s physician is essential for initiating a comprehensive investigation and providing appropriate medical management. The blood bag with attached administration set and a post-transfusion blood sample from the patient must be sent to the blood bank for further investigation, which includes repeat ABO/Rh typing, direct antiglobulin test (DAT), and antibody screening to determine the cause of the reaction. Documenting the transfusion reaction meticulously in the patient’s medical record is vital for future reference and quality improvement purposes. Delaying the investigation or restarting the transfusion before the cause is identified could have severe consequences for the patient.
Incorrect
The correct course of action involves prioritizing the immediate safety of the patient while adhering to regulatory requirements for transfusion reaction investigations. The initial step is to immediately stop the transfusion to prevent further exposure to the potentially harmful component. Maintaining the IV line with normal saline ensures venous access for potential emergency medication administration. A thorough clerical check of patient identification, blood product compatibility, and transfusion paperwork is crucial to identify any errors that may have led to the reaction. Notifying the transfusion service and the patient’s physician is essential for initiating a comprehensive investigation and providing appropriate medical management. The blood bag with attached administration set and a post-transfusion blood sample from the patient must be sent to the blood bank for further investigation, which includes repeat ABO/Rh typing, direct antiglobulin test (DAT), and antibody screening to determine the cause of the reaction. Documenting the transfusion reaction meticulously in the patient’s medical record is vital for future reference and quality improvement purposes. Delaying the investigation or restarting the transfusion before the cause is identified could have severe consequences for the patient.
-
Question 5 of 30
5. Question
Dr. Anya Sharma, the newly appointed medical director of a regional blood bank, is reviewing a complex case involving a pregnant patient, Ms. Isabella Rossi, who is initially typed as group O, D-negative. Ms. Rossi’s obstetrician requests RhIg prophylaxis at 28 weeks gestation. However, during routine pre-transfusion testing before RhIg administration, an unexpected antibody is detected in Ms. Rossi’s serum. Further investigation reveals that Ms. Rossi has no prior history of transfusions or pregnancies, and her initial ABO/Rh typing was performed at an outside facility. The blood bank’s quality assurance records show no recent issues with reagents or equipment. Given this scenario, which of the following actions should Dr. Sharma prioritize to ensure Ms. Rossi receives appropriate and safe care, adhering to both AABB standards and FDA regulations?
Correct
The correct course of action involves several steps, beginning with verifying the reported group O, D-negative status. This is crucial because an incorrect initial typing could lead to the administration of incompatible blood. A thorough review of the patient’s historical blood bank records is necessary to identify any past discrepancies or unusual findings. If no historical data is available or if discrepancies are found, a complete retyping of the patient’s sample, including both forward and reverse typing, should be performed. Furthermore, a weak D test should be conducted to rule out a weak D phenotype, which could be misinterpreted as D-negative. In cases of suspected transfusion reactions, a direct antiglobulin test (DAT) should be performed to detect antibody-coated red cells, indicating an immune-mediated hemolytic process. Antibody screening should be conducted to identify any unexpected alloantibodies that could be causing the incompatibility. If the antibody screen is positive, antibody identification techniques, such as panel studies and enzyme enhancement, should be employed to determine the specificity of the antibody. Once the antibody is identified, antigen typing of the patient’s red cells can confirm the presence or absence of the corresponding antigen. In situations where the patient requires immediate transfusion, group O, D-negative red blood cells should be selected, but only after the initial typing has been verified and potential antibodies have been ruled out or identified. This approach ensures patient safety and minimizes the risk of adverse transfusion reactions, adhering to AABB standards and FDA regulations for blood banking practices.
Incorrect
The correct course of action involves several steps, beginning with verifying the reported group O, D-negative status. This is crucial because an incorrect initial typing could lead to the administration of incompatible blood. A thorough review of the patient’s historical blood bank records is necessary to identify any past discrepancies or unusual findings. If no historical data is available or if discrepancies are found, a complete retyping of the patient’s sample, including both forward and reverse typing, should be performed. Furthermore, a weak D test should be conducted to rule out a weak D phenotype, which could be misinterpreted as D-negative. In cases of suspected transfusion reactions, a direct antiglobulin test (DAT) should be performed to detect antibody-coated red cells, indicating an immune-mediated hemolytic process. Antibody screening should be conducted to identify any unexpected alloantibodies that could be causing the incompatibility. If the antibody screen is positive, antibody identification techniques, such as panel studies and enzyme enhancement, should be employed to determine the specificity of the antibody. Once the antibody is identified, antigen typing of the patient’s red cells can confirm the presence or absence of the corresponding antigen. In situations where the patient requires immediate transfusion, group O, D-negative red blood cells should be selected, but only after the initial typing has been verified and potential antibodies have been ruled out or identified. This approach ensures patient safety and minimizes the risk of adverse transfusion reactions, adhering to AABB standards and FDA regulations for blood banking practices.
-
Question 6 of 30
6. Question
A pregnant patient, Ms. Anya Petrova, who is Rh-negative, undergoes a Kleihauer-Betke test following a suspected placental abruption at 30 weeks gestation. The Kleihauer-Betke test reveals that 0.6% of her red blood cells are fetal cells. Assuming Ms. Petrova has a standard maternal blood volume of 5000 mL, calculate the number of 300 μg RhIg vials required to provide adequate prophylaxis against Rh alloimmunization. According to AABB standards and FDA regulations, the calculated dose should always be rounded up to the nearest whole vial to ensure complete neutralization of fetal Rh-positive red blood cells in the maternal circulation. What is the minimum number of RhIg vials Ms. Petrova should receive?
Correct
To calculate the required dose of RhIg, we first determine the volume of fetal blood in the maternal circulation using the Kleihauer-Betke test results. The formula to estimate the volume of fetal blood is:
Fetal cells (%) * Maternal blood volume (mL) = Volume of fetal blood (mL)
Given:
Fetal cells (%) = 0.6%
Maternal blood volume = 5000 mL (assumed standard maternal blood volume)Volume of fetal blood = \(0.006 \times 5000 = 30\) mL
Next, we calculate the number of RhIg vials needed. One standard dose (300 μg) of RhIg can neutralize 30 mL of Rh-positive whole blood. To determine the number of vials needed, we use the following formula:
Number of vials = Volume of fetal blood (mL) / 30 mL per vial
Number of vials = \(30 / 30 = 1\)
Since the calculated number of vials is exactly 1, we round up to the nearest whole number as per standard practice to ensure adequate coverage. In this case, we don’t need to round up. However, if the result were, for example, 1.2 vials, it would be rounded up to 2 vials. Finally, it’s crucial to confirm the calculated dose with institutional guidelines and consider any additional factors that might influence RhIg dosage. These calculations are essential to prevent Rh alloimmunization in Rh-negative mothers carrying Rh-positive fetuses, a critical aspect of prenatal care and blood banking practices. The importance of accurate calculation and appropriate administration of RhIg cannot be overstated, as it directly impacts the prevention of Hemolytic Disease of the Fetus and Newborn (HDFN).
Incorrect
To calculate the required dose of RhIg, we first determine the volume of fetal blood in the maternal circulation using the Kleihauer-Betke test results. The formula to estimate the volume of fetal blood is:
Fetal cells (%) * Maternal blood volume (mL) = Volume of fetal blood (mL)
Given:
Fetal cells (%) = 0.6%
Maternal blood volume = 5000 mL (assumed standard maternal blood volume)Volume of fetal blood = \(0.006 \times 5000 = 30\) mL
Next, we calculate the number of RhIg vials needed. One standard dose (300 μg) of RhIg can neutralize 30 mL of Rh-positive whole blood. To determine the number of vials needed, we use the following formula:
Number of vials = Volume of fetal blood (mL) / 30 mL per vial
Number of vials = \(30 / 30 = 1\)
Since the calculated number of vials is exactly 1, we round up to the nearest whole number as per standard practice to ensure adequate coverage. In this case, we don’t need to round up. However, if the result were, for example, 1.2 vials, it would be rounded up to 2 vials. Finally, it’s crucial to confirm the calculated dose with institutional guidelines and consider any additional factors that might influence RhIg dosage. These calculations are essential to prevent Rh alloimmunization in Rh-negative mothers carrying Rh-positive fetuses, a critical aspect of prenatal care and blood banking practices. The importance of accurate calculation and appropriate administration of RhIg cannot be overstated, as it directly impacts the prevention of Hemolytic Disease of the Fetus and Newborn (HDFN).
-
Question 7 of 30
7. Question
A 68-year-old female patient with a history of gastrointestinal bleeding requires a two-unit packed red blood cell transfusion. Her blood type is determined to be group O, Rh-positive. The antibody screen is positive, and antibody identification reveals a weakly reactive anti-M. The patient’s red cells type as M+N+. Given the antibody specificity and the patient’s antigen profile, which of the following red cell units would be the MOST appropriate selection for transfusion to minimize the risk of a hemolytic transfusion reaction, considering the potential for antibody dosage effects, even if immediate spin crossmatch is compatible? Assume all units are crossmatch compatible by immediate spin.
Correct
The key to this question lies in understanding the potential for dosage effects in individuals with certain blood group genotypes, particularly concerning the MNSs system and the anti-M antibody. Dosage refers to the phenomenon where antibodies react more strongly with red cells possessing a double dose (homozygous) of the corresponding antigen compared to cells with a single dose (heterozygous). In this case, the patient is group O positive, which eliminates ABO discrepancies. The antibody screen is positive, and the panel reveals a weakly reactive anti-M.
The patient’s red cells type as M+N+. This means they are heterozygous for the M antigen (MN genotype). Anti-M typically shows dosage, meaning it reacts more strongly with MM cells than with MN cells. Since the patient is MN, their cells possess a single dose of the M antigen. The weak reactivity observed in the panel is consistent with this dosage effect.
If transfused with M+N- red cells, the patient would be receiving red cells that are homozygous for the M antigen (MM genotype). Because anti-M exhibits dosage, the antibody would react more strongly with these transfused cells than with the patient’s own MN cells. This could lead to an increased risk of a hemolytic transfusion reaction, even if the initial crossmatch was compatible at immediate spin. The use of M-N+ or M-N- red cells is preferable as they lack the M antigen entirely, thus avoiding the potential for a stronger reaction. The decision to transfuse is based on clinical need and the potential risk of the antibody.
Incorrect
The key to this question lies in understanding the potential for dosage effects in individuals with certain blood group genotypes, particularly concerning the MNSs system and the anti-M antibody. Dosage refers to the phenomenon where antibodies react more strongly with red cells possessing a double dose (homozygous) of the corresponding antigen compared to cells with a single dose (heterozygous). In this case, the patient is group O positive, which eliminates ABO discrepancies. The antibody screen is positive, and the panel reveals a weakly reactive anti-M.
The patient’s red cells type as M+N+. This means they are heterozygous for the M antigen (MN genotype). Anti-M typically shows dosage, meaning it reacts more strongly with MM cells than with MN cells. Since the patient is MN, their cells possess a single dose of the M antigen. The weak reactivity observed in the panel is consistent with this dosage effect.
If transfused with M+N- red cells, the patient would be receiving red cells that are homozygous for the M antigen (MM genotype). Because anti-M exhibits dosage, the antibody would react more strongly with these transfused cells than with the patient’s own MN cells. This could lead to an increased risk of a hemolytic transfusion reaction, even if the initial crossmatch was compatible at immediate spin. The use of M-N+ or M-N- red cells is preferable as they lack the M antigen entirely, thus avoiding the potential for a stronger reaction. The decision to transfuse is based on clinical need and the potential risk of the antibody.
-
Question 8 of 30
8. Question
Dr. Anya Sharma, a transfusion medicine specialist, is consulted on a complex case at County General Hospital. Her patient, Mr. Jian Li, a 68-year-old male with a history of autoimmune hemolytic anemia, requires an urgent blood transfusion due to a significant drop in hemoglobin following a recent surgical procedure. Mr. Li’s pre-transfusion workup reveals a strongly positive direct antiglobulin test (DAT) and a pan-reactive antibody screen. Initial attempts at antibody identification are inconclusive due to the presence of a warm autoantibody. The blood bank technologist, recognizing the complexity of the case, seeks Dr. Sharma’s expertise to ensure the safest possible transfusion strategy for Mr. Li, considering the potential risks of alloantibody masking and delayed hemolytic transfusion reaction. Given this scenario, what is the MOST appropriate initial step Dr. Sharma should recommend to the blood bank technologist to facilitate accurate blood typing and compatibility testing for Mr. Li?
Correct
The key to managing a patient with a warm autoantibody lies in understanding the challenges it presents for accurate blood typing and compatibility testing. Warm autoantibodies react at 37°C and can mask underlying alloantibodies, leading to inaccurate crossmatch results and potentially causing a delayed hemolytic transfusion reaction if incompatible blood is transfused.
First, perform an auto control and DAT (Direct Antiglobulin Test). A positive DAT indicates that the patient’s red cells are coated with IgG and/or complement. Next, perform an elution to remove the autoantibody from the red cells, allowing for accurate antigen typing of the patient’s red cells. The eluate should then be tested against a panel of reagent red cells to determine if an underlying alloantibody is present in addition to the autoantibody. If an alloantibody is identified, antigen-negative units should be selected for transfusion. If no alloantibody is identified, select the least incompatible units for transfusion.
In cases where compatible units cannot be found, strategies such as using autologous blood, if available, or administering immunosuppressive therapy to reduce the autoantibody titer may be considered. Close monitoring for signs of a transfusion reaction is crucial in patients with warm autoantibodies.
Incorrect
The key to managing a patient with a warm autoantibody lies in understanding the challenges it presents for accurate blood typing and compatibility testing. Warm autoantibodies react at 37°C and can mask underlying alloantibodies, leading to inaccurate crossmatch results and potentially causing a delayed hemolytic transfusion reaction if incompatible blood is transfused.
First, perform an auto control and DAT (Direct Antiglobulin Test). A positive DAT indicates that the patient’s red cells are coated with IgG and/or complement. Next, perform an elution to remove the autoantibody from the red cells, allowing for accurate antigen typing of the patient’s red cells. The eluate should then be tested against a panel of reagent red cells to determine if an underlying alloantibody is present in addition to the autoantibody. If an alloantibody is identified, antigen-negative units should be selected for transfusion. If no alloantibody is identified, select the least incompatible units for transfusion.
In cases where compatible units cannot be found, strategies such as using autologous blood, if available, or administering immunosuppressive therapy to reduce the autoantibody titer may be considered. Close monitoring for signs of a transfusion reaction is crucial in patients with warm autoantibodies.
-
Question 9 of 30
9. Question
A 70 kg patient with severe thrombocytopenia requires platelet transfusion to increase their platelet count by \(25,000/\mu L\). The blood bank uses platelet concentrates that contain an average of \(5.5 \times 10^{10}\) platelets per unit. According to AABB standards and hospital transfusion protocols, how many platelet concentrates are required to achieve the desired platelet increment, rounding up to the nearest whole number if necessary, to ensure adequate hemostasis and minimize the risk of spontaneous bleeding, taking into account the patient’s weight and the typical platelet yield from each concentrate?
Correct
The calculation of the required number of platelet concentrates involves several steps. First, determine the required platelet increase: \(25,000/\mu L\). Then, calculate the platelet increment needed: \(25,000/\mu L \times 70 kg = 1,750,000 \times 10^{11}\) platelets. Next, determine the number of platelets per concentrate: \(5.5 \times 10^{10}\) platelets. Finally, divide the total platelets needed by the platelets per concentrate: \(\frac{1.75 \times 10^{11}}{5.5 \times 10^{10}} = 3.18\). Since you can’t administer a fraction of a platelet concentrate, round up to the nearest whole number. Therefore, 4 platelet concentrates are needed.
The ABO blood group system is critical in transfusion medicine because of the presence of naturally occurring antibodies. Incompatibility can lead to acute hemolytic transfusion reactions, which can be life-threatening. Understanding the dosage of platelet concentrates is crucial to ensure adequate hemostasis and prevent bleeding complications. In this scenario, accurately calculating the number of platelet concentrates is essential for managing a patient with thrombocytopenia. Errors in calculation can lead to under- or over-transfusion, both of which can have adverse clinical outcomes. The calculation must consider the patient’s weight and the expected platelet increment from each concentrate. This ensures that the patient receives an adequate dose of platelets to achieve the desired therapeutic effect while minimizing the risk of transfusion-related complications. Correct calculation is a fundamental aspect of transfusion safety and patient care.
Incorrect
The calculation of the required number of platelet concentrates involves several steps. First, determine the required platelet increase: \(25,000/\mu L\). Then, calculate the platelet increment needed: \(25,000/\mu L \times 70 kg = 1,750,000 \times 10^{11}\) platelets. Next, determine the number of platelets per concentrate: \(5.5 \times 10^{10}\) platelets. Finally, divide the total platelets needed by the platelets per concentrate: \(\frac{1.75 \times 10^{11}}{5.5 \times 10^{10}} = 3.18\). Since you can’t administer a fraction of a platelet concentrate, round up to the nearest whole number. Therefore, 4 platelet concentrates are needed.
The ABO blood group system is critical in transfusion medicine because of the presence of naturally occurring antibodies. Incompatibility can lead to acute hemolytic transfusion reactions, which can be life-threatening. Understanding the dosage of platelet concentrates is crucial to ensure adequate hemostasis and prevent bleeding complications. In this scenario, accurately calculating the number of platelet concentrates is essential for managing a patient with thrombocytopenia. Errors in calculation can lead to under- or over-transfusion, both of which can have adverse clinical outcomes. The calculation must consider the patient’s weight and the expected platelet increment from each concentrate. This ensures that the patient receives an adequate dose of platelets to achieve the desired therapeutic effect while minimizing the risk of transfusion-related complications. Correct calculation is a fundamental aspect of transfusion safety and patient care.
-
Question 10 of 30
10. Question
A neonate, born to a group O Rh-positive mother, is suspected of having ABO hemolytic disease of the fetus and newborn (HDFN). The neonate’s initial ABO typing shows a forward type A and a weak, mixed-field reaction in the reverse grouping. The direct antiglobulin test (DAT) is positive. Dr. Ramirez requests a transfusion of packed red blood cells for the neonate. Considering the immunohematologic findings and the clinical context, what is the MOST appropriate course of action for the blood bank to take to ensure the safety and efficacy of the transfusion? Explain the rationale behind each step.
Correct
The scenario describes a situation where the initial ABO typing of a neonate conflicts with the mother’s known blood type, and the direct antiglobulin test (DAT) is positive. This strongly suggests ABO hemolytic disease of the fetus and newborn (HDFN), particularly if the mother is group O and the infant is group A or B. Group O individuals possess anti-A, anti-B, and anti-A,B IgG antibodies, which can cross the placenta and react with fetal red cells expressing the corresponding antigen. The DAT detects antibody (in this case, likely anti-A or anti-B) coating the infant’s red cells. A weak or mixed-field reaction in the reverse grouping of the neonate is expected because the neonate’s antibody production is not yet mature, and any passively acquired maternal antibodies are being diluted out and/or consumed by reacting with the infant’s own cells. Elution studies would be performed to identify the antibody coating the infant’s red cells; in this case, anti-A or anti-B would be expected. The blood bank should select group O, Rh-compatible red blood cells for transfusion. These cells lack A and B antigens, preventing further antibody-antigen reactions and exacerbation of the hemolysis. The selected RBCs should be CMV negative and irradiated to prevent CMV transmission and transfusion-associated graft-versus-host disease (TA-GVHD) in the immunocompromised neonate. The units must be compatible by crossmatch with the mother’s serum or an eluate of the infant’s red cells.
Incorrect
The scenario describes a situation where the initial ABO typing of a neonate conflicts with the mother’s known blood type, and the direct antiglobulin test (DAT) is positive. This strongly suggests ABO hemolytic disease of the fetus and newborn (HDFN), particularly if the mother is group O and the infant is group A or B. Group O individuals possess anti-A, anti-B, and anti-A,B IgG antibodies, which can cross the placenta and react with fetal red cells expressing the corresponding antigen. The DAT detects antibody (in this case, likely anti-A or anti-B) coating the infant’s red cells. A weak or mixed-field reaction in the reverse grouping of the neonate is expected because the neonate’s antibody production is not yet mature, and any passively acquired maternal antibodies are being diluted out and/or consumed by reacting with the infant’s own cells. Elution studies would be performed to identify the antibody coating the infant’s red cells; in this case, anti-A or anti-B would be expected. The blood bank should select group O, Rh-compatible red blood cells for transfusion. These cells lack A and B antigens, preventing further antibody-antigen reactions and exacerbation of the hemolysis. The selected RBCs should be CMV negative and irradiated to prevent CMV transmission and transfusion-associated graft-versus-host disease (TA-GVHD) in the immunocompromised neonate. The units must be compatible by crossmatch with the mother’s serum or an eluate of the infant’s red cells.
-
Question 11 of 30
11. Question
A 62-year-old patient, Mr. Jian, with a history of multiple red blood cell transfusions due to chronic anemia, presents to the blood bank for pre-transfusion testing. Antibody screening is positive. The antibody identification panel results show reactivity with all panel cells tested except for one. The autocontrol is negative. Further investigation reveals that the non-reactive panel cell has the following antigen profile: R1R1, K-, Jka-, Fya-. Given this information and assuming all common alloantibodies have been ruled out, what is the most probable antibody specificity causing the observed reactivity pattern? Consider the prevalence and clinical significance of various blood group antibodies in multiple-transfused patients.
Correct
The scenario describes a situation where a patient with a history of multiple transfusions develops a new antibody that reacts with all panel cells except for one. The autocontrol is negative, indicating the antibody is likely alloantibody and not autoantibody. The single non-reactive cell is R1R1, K-, Jka-, Fya-. To determine the most probable antibody specificity, we need to consider which common high-frequency antigens are missing on that cell. The most likely antibody is anti-Kpb. Kpb is a high-frequency antigen within the Kell blood group system. The other options are less likely because their corresponding antigens are not high-frequency. The scenario emphasizes a high-frequency antigen, making anti-Kpb the most probable answer. Anti-Lea is not typically considered clinically significant in transfusion. Anti-Yta is also a high-frequency antigen, but the scenario does not explicitly mention ruling out other high-frequency antibodies before focusing on the Kell system. The exclusion of other common antibodies like anti-e, anti-Cellano (k), or anti-Vel is crucial in such investigations. Therefore, considering the information provided, the most plausible antibody is anti-Kpb.
Incorrect
The scenario describes a situation where a patient with a history of multiple transfusions develops a new antibody that reacts with all panel cells except for one. The autocontrol is negative, indicating the antibody is likely alloantibody and not autoantibody. The single non-reactive cell is R1R1, K-, Jka-, Fya-. To determine the most probable antibody specificity, we need to consider which common high-frequency antigens are missing on that cell. The most likely antibody is anti-Kpb. Kpb is a high-frequency antigen within the Kell blood group system. The other options are less likely because their corresponding antigens are not high-frequency. The scenario emphasizes a high-frequency antigen, making anti-Kpb the most probable answer. Anti-Lea is not typically considered clinically significant in transfusion. Anti-Yta is also a high-frequency antigen, but the scenario does not explicitly mention ruling out other high-frequency antibodies before focusing on the Kell system. The exclusion of other common antibodies like anti-e, anti-Cellano (k), or anti-Vel is crucial in such investigations. Therefore, considering the information provided, the most plausible antibody is anti-Kpb.
-
Question 12 of 30
12. Question
Dr. Anya Sharma, a transfusion medicine specialist, is consulted for a patient, Mr. Jian Li, who is thrombocytopenic and requires platelet transfusion support. Mr. Li has a body surface area (BSA) of \(1.8 m^2\) and needs his platelet count increased by \(50,000/\mu L\). The hospital’s standard operating procedure (SOP) indicates that the expected platelet recovery in this patient population is approximately 60%. Each plateletpheresis unit available in the blood bank contains \(3 \times 10^{11}\) platelets.
Using the formula:
\[ \text{Number of Plateletpheresis Units} = \frac{\text{Desired Increment} \times \text{BSA} \times \text{Platelet Count Conversion Factor}}{\text{Platelets per Unit} \times \text{Recovery}} \]
and assuming a platelet count conversion factor of \(1 \times 10^{11}\), how many plateletpheresis units are required to achieve the desired platelet increment for Mr. Li, taking into account his BSA and the expected platelet recovery?
Correct
To calculate the required number of platelets, we need to determine the platelet increment needed and then account for platelet recovery and volume. The desired platelet increment is \( 50,000/\mu L \). The patient’s body surface area (BSA) is \( 1.8 m^2 \). The standard formula to calculate the number of platelets needed is:
\[ \text{Platelet Increment} = \text{Desired Increment} \times \text{BSA} \times \text{Platelet Count Conversion Factor} \]
Assuming the platelet count conversion factor is 10^11 platelets/\(\mu L\)/m^2:
\[ \text{Platelets Needed} = 50,000/\mu L \times 1.8 m^2 \times 10^{11} \text{ platelets}/\mu L/m^2 = 9 \times 10^{15} \text{ platelets} \]
Now, we must account for platelet recovery. If the expected recovery is 60%, then we need to adjust the platelet count to compensate for the loss:
\[ \text{Adjusted Platelets Needed} = \frac{\text{Platelets Needed}}{\text{Recovery}} = \frac{9 \times 10^{15}}{0.6} = 1.5 \times 10^{16} \text{ platelets} \]
Next, we need to determine the number of plateletpheresis units required. If each unit contains \( 3 \times 10^{11} \) platelets, then:
\[ \text{Number of Units} = \frac{\text{Adjusted Platelets Needed}}{\text{Platelets per Unit}} = \frac{1.5 \times 10^{16}}{3 \times 10^{11}} = 50,000 \text{ units} \]
However, this result seems extremely high, indicating an error in the assumed platelet count conversion factor. A more reasonable conversion factor is \( 1 \times 10^{11} \). Let’s recalculate using this factor:
\[ \text{Platelets Needed} = 50,000/\mu L \times 1.8 m^2 \times 1 \times 10^{11} \text{ platelets}/\mu L/m^2 = 9 \times 10^{15} \text{ platelets} \]
\[ \text{Adjusted Platelets Needed} = \frac{9 \times 10^{15}}{0.6} = 1.5 \times 10^{16} \text{ platelets} \]
\[ \text{Number of Units} = \frac{1.5 \times 10^{12}}{3 \times 10^{11}} = 5 \text{ units} \]
Therefore, 5 plateletpheresis units are required to achieve the desired platelet increment, considering the patient’s BSA and expected platelet recovery. The critical point is understanding the relationship between desired increment, BSA, platelet recovery, and the platelet content per unit, as well as using appropriate conversion factors.
Incorrect
To calculate the required number of platelets, we need to determine the platelet increment needed and then account for platelet recovery and volume. The desired platelet increment is \( 50,000/\mu L \). The patient’s body surface area (BSA) is \( 1.8 m^2 \). The standard formula to calculate the number of platelets needed is:
\[ \text{Platelet Increment} = \text{Desired Increment} \times \text{BSA} \times \text{Platelet Count Conversion Factor} \]
Assuming the platelet count conversion factor is 10^11 platelets/\(\mu L\)/m^2:
\[ \text{Platelets Needed} = 50,000/\mu L \times 1.8 m^2 \times 10^{11} \text{ platelets}/\mu L/m^2 = 9 \times 10^{15} \text{ platelets} \]
Now, we must account for platelet recovery. If the expected recovery is 60%, then we need to adjust the platelet count to compensate for the loss:
\[ \text{Adjusted Platelets Needed} = \frac{\text{Platelets Needed}}{\text{Recovery}} = \frac{9 \times 10^{15}}{0.6} = 1.5 \times 10^{16} \text{ platelets} \]
Next, we need to determine the number of plateletpheresis units required. If each unit contains \( 3 \times 10^{11} \) platelets, then:
\[ \text{Number of Units} = \frac{\text{Adjusted Platelets Needed}}{\text{Platelets per Unit}} = \frac{1.5 \times 10^{16}}{3 \times 10^{11}} = 50,000 \text{ units} \]
However, this result seems extremely high, indicating an error in the assumed platelet count conversion factor. A more reasonable conversion factor is \( 1 \times 10^{11} \). Let’s recalculate using this factor:
\[ \text{Platelets Needed} = 50,000/\mu L \times 1.8 m^2 \times 1 \times 10^{11} \text{ platelets}/\mu L/m^2 = 9 \times 10^{15} \text{ platelets} \]
\[ \text{Adjusted Platelets Needed} = \frac{9 \times 10^{15}}{0.6} = 1.5 \times 10^{16} \text{ platelets} \]
\[ \text{Number of Units} = \frac{1.5 \times 10^{12}}{3 \times 10^{11}} = 5 \text{ units} \]
Therefore, 5 plateletpheresis units are required to achieve the desired platelet increment, considering the patient’s BSA and expected platelet recovery. The critical point is understanding the relationship between desired increment, BSA, platelet recovery, and the platelet content per unit, as well as using appropriate conversion factors.
-
Question 13 of 30
13. Question
Dr. Ramirez, a blood bank specialist, is consulted on a complex case involving a 68-year-old patient, Ms. Ishikawa, with a history of myelodysplastic syndrome (MDS) undergoing pre-transfusion testing. Initial ABO typing reveals a weak and inconsistent forward grouping reaction with anti-A reagent (grade 1+), while the reverse grouping shows an apparent anti-B reactivity that does not correlate with her presumed blood type based on historical records. Repeat testing yields similar results, and the possibility of a technical error has been ruled out. Further investigation reveals that Ms. Ishikawa has not received any recent transfusions. Considering the patient’s medical history and the serological findings, which of the following is the MOST appropriate next step in resolving this ABO discrepancy and ensuring the selection of compatible blood products?
Correct
The question explores the complexities surrounding ABO discrepancies, specifically focusing on individuals exhibiting weak or unusual ABO phenotypes and the potential underlying causes. The correct approach involves considering various factors that can influence ABO expression, including genetic mutations, disease states, and technical errors. A thorough investigation is crucial to accurately determine the patient’s blood type and ensure safe transfusion practices. The scenario highlights the importance of employing a comprehensive testing strategy, including repeat testing with different reagent lots, incubation at lower temperatures (4°C) to enhance weak reactions, adsorption-elution studies to identify weakly expressed antigens, and family studies to clarify inheritance patterns. Furthermore, it is vital to rule out potential causes such as leukemia, which can sometimes suppress the expression of ABO antigens. The ultimate goal is to resolve the discrepancy and provide compatible blood products while adhering to regulatory guidelines and best practices in immunohematology. Consideration should also be given to the possibility of a cis-AB inheritance, where A and B genes are inherited on the same chromosome, leading to weakened expression. Ruling out acquired B phenomenon (associated with bacterial infections) is also important in certain cases. The explanation should cover these points to ensure a deep understanding of ABO discrepancies and their resolution.
Incorrect
The question explores the complexities surrounding ABO discrepancies, specifically focusing on individuals exhibiting weak or unusual ABO phenotypes and the potential underlying causes. The correct approach involves considering various factors that can influence ABO expression, including genetic mutations, disease states, and technical errors. A thorough investigation is crucial to accurately determine the patient’s blood type and ensure safe transfusion practices. The scenario highlights the importance of employing a comprehensive testing strategy, including repeat testing with different reagent lots, incubation at lower temperatures (4°C) to enhance weak reactions, adsorption-elution studies to identify weakly expressed antigens, and family studies to clarify inheritance patterns. Furthermore, it is vital to rule out potential causes such as leukemia, which can sometimes suppress the expression of ABO antigens. The ultimate goal is to resolve the discrepancy and provide compatible blood products while adhering to regulatory guidelines and best practices in immunohematology. Consideration should also be given to the possibility of a cis-AB inheritance, where A and B genes are inherited on the same chromosome, leading to weakened expression. Ruling out acquired B phenomenon (associated with bacterial infections) is also important in certain cases. The explanation should cover these points to ensure a deep understanding of ABO discrepancies and their resolution.
-
Question 14 of 30
14. Question
A 28-year-old pregnant woman, Leticia, at 20 weeks gestation, presents to her obstetrician with a history of anti-c identified during her previous pregnancy, for which she received RhIg after delivery. Her current antibody screen is positive at the AHG phase using IgG-coated red cells but negative at the immediate spin (IS) phase. Antibody identification confirms the presence of anti-c. The laboratory performs a titer, and the initial titer is 1:4. Leticia has no history of transfusions. According to AABB standards and accepted immunohematology practices, what is the MOST appropriate next step in managing Leticia’s case to assess the risk of Hemolytic Disease of the Fetus and Newborn (HDFN)?
Correct
The scenario presents a complex situation involving a pregnant woman with a history of anti-c and a current antibody screen positive at AHG phase, but negative at IS phase. The key is to differentiate between passively acquired anti-c (likely from RhIg) and actively produced anti-c, which would be more concerning for HDFN. Titration is essential to determine the antibody’s concentration and monitor any increase over time, which would indicate active production. An initial titer of 1:4 is relatively low and could be consistent with passive immunization, but serial titers are crucial. Fetal monitoring via MCA Doppler is indicated if the titer reaches a critical level (typically 1:16 or 1:32, depending on the laboratory’s protocol and the specific antibody). RhIg is contraindicated in this scenario because the patient already has anti-c. Performing an antibody identification is already done in the scenario. Therefore, the most appropriate next step is to perform serial antibody titers to monitor the antibody concentration. The decision to perform MCA Doppler studies will depend on the titer result and gestational age.
Incorrect
The scenario presents a complex situation involving a pregnant woman with a history of anti-c and a current antibody screen positive at AHG phase, but negative at IS phase. The key is to differentiate between passively acquired anti-c (likely from RhIg) and actively produced anti-c, which would be more concerning for HDFN. Titration is essential to determine the antibody’s concentration and monitor any increase over time, which would indicate active production. An initial titer of 1:4 is relatively low and could be consistent with passive immunization, but serial titers are crucial. Fetal monitoring via MCA Doppler is indicated if the titer reaches a critical level (typically 1:16 or 1:32, depending on the laboratory’s protocol and the specific antibody). RhIg is contraindicated in this scenario because the patient already has anti-c. Performing an antibody identification is already done in the scenario. Therefore, the most appropriate next step is to perform serial antibody titers to monitor the antibody concentration. The decision to perform MCA Doppler studies will depend on the titer result and gestational age.
-
Question 15 of 30
15. Question
Dr. Anya Sharma, a hematologist at County General Hospital, requests a blood transfusion for a 70 kg patient, Mr. Jian Li, who has a current hemoglobin (Hb) level of 7 g/dL. Dr. Sharma wants to increase Mr. Li’s Hb to 10 g/dL. Assuming Mr. Li has a normal blood volume for his weight and that the packed red blood cells (PRBCs) available have a hemoglobin concentration of 20 g/dL with a volume of 300 mL per unit, how many units of PRBCs should be transfused to achieve the desired hemoglobin level? (Assume adult blood volume factor is 70 mL/kg). Round up to the nearest whole unit.
Correct
The calculation involves determining the required volume of packed red blood cells (PRBCs) to raise a patient’s hemoglobin (Hb) level to a desired target, considering the patient’s blood volume and the Hb concentration of the PRBCs.
First, calculate the patient’s blood volume (PBV) using the formula:
\[ PBV = Weight (kg) \times Blood Volume Factor (mL/kg) \]
Here, the patient’s weight is 70 kg, and the blood volume factor for adults is approximately 70 mL/kg.
\[ PBV = 70 \, kg \times 70 \, mL/kg = 4900 \, mL \]Next, determine the hemoglobin deficit:
\[ Hb \, Deficit = Desired \, Hb – Current \, Hb \]
\[ Hb \, Deficit = 10 \, g/dL – 7 \, g/dL = 3 \, g/dL \]Then, calculate the total red blood cell mass needed to correct the deficit:
\[ RBC \, Mass \, Needed = PBV (mL) \times Hb \, Deficit (g/dL) \]
\[ RBC \, Mass \, Needed = 4900 \, mL \times 3 \, g/dL = 14700 \, g \cdot mL/dL \]
Convert mL to dL: Since 1 dL = 100 mL,
\[ RBC \, Mass \, Needed = \frac{14700}{100} = 147 \, g \]Now, calculate the volume of PRBCs needed. A typical unit of PRBCs has a Hb concentration of approximately 20 g/dL, and a volume of approximately 300 mL. The actual Hb content in grams is:
\[ Hb \, Content \, per \, PRBC = Concentration \times Volume = 20 \, g/dL \times 300 \, mL \]
Convert mL to dL: Since 1 dL = 100 mL,
\[ Hb \, Content \, per \, PRBC = 20 \, g/dL \times 3 \, dL = 60 \, g \]Finally, calculate the number of PRBC units needed:
\[ Units \, Needed = \frac{RBC \, Mass \, Needed}{Hb \, Content \, per \, PRBC} = \frac{147 \, g}{60 \, g/unit} = 2.45 \, units \]
Since you cannot transfuse a fraction of a unit, round up to the nearest whole number. Therefore, 3 units of PRBCs are needed.This calculation incorporates several crucial blood banking principles. First, accurately estimating the patient’s blood volume is essential for determining the necessary transfusion volume. Second, understanding the hemoglobin concentration of the transfused product is vital for calculating the expected rise in the patient’s hemoglobin level. Third, the calculation underscores the importance of considering the physiological impact of the transfusion, ensuring that the patient receives an adequate but not excessive volume of blood. Finally, the rounding up of the units needed highlights the clinical decision-making process, balancing the need for effective treatment with the risks associated with transfusion. The use of appropriate formulas and conversion factors ensures accurate dosage calculation, a critical aspect of transfusion medicine governed by strict regulatory standards and guidelines.
Incorrect
The calculation involves determining the required volume of packed red blood cells (PRBCs) to raise a patient’s hemoglobin (Hb) level to a desired target, considering the patient’s blood volume and the Hb concentration of the PRBCs.
First, calculate the patient’s blood volume (PBV) using the formula:
\[ PBV = Weight (kg) \times Blood Volume Factor (mL/kg) \]
Here, the patient’s weight is 70 kg, and the blood volume factor for adults is approximately 70 mL/kg.
\[ PBV = 70 \, kg \times 70 \, mL/kg = 4900 \, mL \]Next, determine the hemoglobin deficit:
\[ Hb \, Deficit = Desired \, Hb – Current \, Hb \]
\[ Hb \, Deficit = 10 \, g/dL – 7 \, g/dL = 3 \, g/dL \]Then, calculate the total red blood cell mass needed to correct the deficit:
\[ RBC \, Mass \, Needed = PBV (mL) \times Hb \, Deficit (g/dL) \]
\[ RBC \, Mass \, Needed = 4900 \, mL \times 3 \, g/dL = 14700 \, g \cdot mL/dL \]
Convert mL to dL: Since 1 dL = 100 mL,
\[ RBC \, Mass \, Needed = \frac{14700}{100} = 147 \, g \]Now, calculate the volume of PRBCs needed. A typical unit of PRBCs has a Hb concentration of approximately 20 g/dL, and a volume of approximately 300 mL. The actual Hb content in grams is:
\[ Hb \, Content \, per \, PRBC = Concentration \times Volume = 20 \, g/dL \times 300 \, mL \]
Convert mL to dL: Since 1 dL = 100 mL,
\[ Hb \, Content \, per \, PRBC = 20 \, g/dL \times 3 \, dL = 60 \, g \]Finally, calculate the number of PRBC units needed:
\[ Units \, Needed = \frac{RBC \, Mass \, Needed}{Hb \, Content \, per \, PRBC} = \frac{147 \, g}{60 \, g/unit} = 2.45 \, units \]
Since you cannot transfuse a fraction of a unit, round up to the nearest whole number. Therefore, 3 units of PRBCs are needed.This calculation incorporates several crucial blood banking principles. First, accurately estimating the patient’s blood volume is essential for determining the necessary transfusion volume. Second, understanding the hemoglobin concentration of the transfused product is vital for calculating the expected rise in the patient’s hemoglobin level. Third, the calculation underscores the importance of considering the physiological impact of the transfusion, ensuring that the patient receives an adequate but not excessive volume of blood. Finally, the rounding up of the units needed highlights the clinical decision-making process, balancing the need for effective treatment with the risks associated with transfusion. The use of appropriate formulas and conversion factors ensures accurate dosage calculation, a critical aspect of transfusion medicine governed by strict regulatory standards and guidelines.
-
Question 16 of 30
16. Question
A 28-year-old pregnant woman, Leticia, at 24 weeks gestation, presents to her obstetrician. Her prenatal screening reveals a history of anti-E and a newly identified anti-c antibody. Her antibody titers for anti-c are significantly elevated. The obstetrician consults with the blood bank medical director, Dr. Anya Sharma, to determine the best course of action. Leticia’s husband, Javier, undergoes blood typing, and his results are pending. Dr. Sharma knows the importance of preventing Hemolytic Disease of the Fetus and Newborn (HDFN). Considering the patient’s history and current presentation, which of the following is the MOST appropriate next step in managing Leticia’s pregnancy to assess fetal risk?
Correct
The scenario describes a complex situation involving a pregnant patient with a history of anti-E and now presenting with an anti-c. The key is to understand the potential for HDFN and the appropriate steps to ensure the fetus’s safety. First, confirm the antibody specificities and titers. A high titer of anti-c, especially in conjunction with a pre-existing anti-E, significantly increases the risk of HDFN. The next crucial step is to determine the paternal antigen status for both c and E. If the father is negative for both antigens, the fetus is also negative, and no further intervention is needed beyond routine monitoring. If the father is positive for either or both antigens, fetal antigen typing becomes critical. Amniocentesis or cell-free fetal DNA testing can determine if the fetus possesses the c antigen. If the fetus is c-positive, serial antibody titers via maternal blood samples are essential to monitor the severity of the antibody response. Doppler velocimetry, specifically measuring the middle cerebral artery peak systolic velocity (MCA-PSV), is the gold standard for detecting fetal anemia non-invasively. An elevated MCA-PSV indicates fetal anemia, prompting consideration of intrauterine transfusion. If the MCA-PSV is normal, monitoring continues, but if it’s elevated, cordocentesis (fetal blood sampling) is performed to directly assess the fetal hemoglobin level and, if necessary, administer an intrauterine transfusion. The decision to perform an intrauterine transfusion is based on the fetal hemoglobin level and gestational age, aiming to prevent hydrops fetalis and ensure fetal survival. RhIG is not indicated in this case because anti-c is not an anti-D antibody, and the patient is already alloimmunized.
Incorrect
The scenario describes a complex situation involving a pregnant patient with a history of anti-E and now presenting with an anti-c. The key is to understand the potential for HDFN and the appropriate steps to ensure the fetus’s safety. First, confirm the antibody specificities and titers. A high titer of anti-c, especially in conjunction with a pre-existing anti-E, significantly increases the risk of HDFN. The next crucial step is to determine the paternal antigen status for both c and E. If the father is negative for both antigens, the fetus is also negative, and no further intervention is needed beyond routine monitoring. If the father is positive for either or both antigens, fetal antigen typing becomes critical. Amniocentesis or cell-free fetal DNA testing can determine if the fetus possesses the c antigen. If the fetus is c-positive, serial antibody titers via maternal blood samples are essential to monitor the severity of the antibody response. Doppler velocimetry, specifically measuring the middle cerebral artery peak systolic velocity (MCA-PSV), is the gold standard for detecting fetal anemia non-invasively. An elevated MCA-PSV indicates fetal anemia, prompting consideration of intrauterine transfusion. If the MCA-PSV is normal, monitoring continues, but if it’s elevated, cordocentesis (fetal blood sampling) is performed to directly assess the fetal hemoglobin level and, if necessary, administer an intrauterine transfusion. The decision to perform an intrauterine transfusion is based on the fetal hemoglobin level and gestational age, aiming to prevent hydrops fetalis and ensure fetal survival. RhIG is not indicated in this case because anti-c is not an anti-D antibody, and the patient is already alloimmunized.
-
Question 17 of 30
17. Question
A 62-year-old female patient, Leticia Rodriguez, with a history of multiple transfusions due to chronic anemia, presents to the blood bank for pre-transfusion testing. Initial antibody screening reveals a positive result with a 2+ reaction at the AHG phase in all panel cells tested. The autocontrol is also positive (2+). An initial antibody identification panel suggests the presence of anti-e, but the reactivity pattern is inconsistent, with some e-positive cells reacting weakly or not at all. Suspecting a warm autoantibody masking an underlying alloantibody, the blood bank scientist performs differential adsorption studies. One aliquot of Leticia’s serum is adsorbed with R1R1 cells, and another aliquot is adsorbed with R2R2 cells. After adsorption, each serum aliquot is tested against an antibody identification panel. The R1R1 adsorbed serum shows significantly reduced reactivity against cells expressing the E antigen but not against cells expressing the c antigen. Conversely, the R2R2 adsorbed serum shows significantly reduced reactivity against cells expressing the c antigen but not against cells expressing the E antigen. Elution studies performed on the patient’s red cells confirm the presence of a warm autoantibody with broad specificity. Based on these findings, what are the most likely alloantibodies present in Leticia Rodriguez’s serum, in addition to the autoantibody?
Correct
The scenario describes a complex situation involving a patient with a warm autoantibody masking an underlying alloantibody. The initial workup suggests anti-e, but the reactivity pattern is inconsistent, and the autocontrol is positive, indicating the presence of an autoantibody. Adsorption techniques are used to remove the autoantibody, revealing the presence of an underlying alloantibody. The key to identifying the alloantibody is understanding the principle of differential adsorption. In differential adsorption, multiple aliquots of the patient’s serum are adsorbed with different red cell phenotypes. In this case, one aliquot is adsorbed with R1R1 cells (D positive, C positive, E negative, c negative, e positive), and another is adsorbed with R2R2 cells (D positive, C negative, E positive, c positive, e negative). After adsorption, each serum aliquot is tested against a panel of red cells. If the R1R1 adsorbed serum shows reduced reactivity against cells expressing E antigen but not against cells expressing c antigen, and the R2R2 adsorbed serum shows reduced reactivity against cells expressing c antigen but not against cells expressing E antigen, it suggests the presence of both anti-E and anti-c. The elution studies confirm the presence of the autoantibody, while the adsorption studies help to identify the underlying alloantibodies. Therefore, the most likely alloantibodies present in the patient’s serum are anti-E and anti-c. This approach is crucial in resolving complex antibody problems and ensuring the selection of compatible blood for transfusion.
Incorrect
The scenario describes a complex situation involving a patient with a warm autoantibody masking an underlying alloantibody. The initial workup suggests anti-e, but the reactivity pattern is inconsistent, and the autocontrol is positive, indicating the presence of an autoantibody. Adsorption techniques are used to remove the autoantibody, revealing the presence of an underlying alloantibody. The key to identifying the alloantibody is understanding the principle of differential adsorption. In differential adsorption, multiple aliquots of the patient’s serum are adsorbed with different red cell phenotypes. In this case, one aliquot is adsorbed with R1R1 cells (D positive, C positive, E negative, c negative, e positive), and another is adsorbed with R2R2 cells (D positive, C negative, E positive, c positive, e negative). After adsorption, each serum aliquot is tested against a panel of red cells. If the R1R1 adsorbed serum shows reduced reactivity against cells expressing E antigen but not against cells expressing c antigen, and the R2R2 adsorbed serum shows reduced reactivity against cells expressing c antigen but not against cells expressing E antigen, it suggests the presence of both anti-E and anti-c. The elution studies confirm the presence of the autoantibody, while the adsorption studies help to identify the underlying alloantibodies. Therefore, the most likely alloantibodies present in the patient’s serum are anti-E and anti-c. This approach is crucial in resolving complex antibody problems and ensuring the selection of compatible blood for transfusion.
-
Question 18 of 30
18. Question
Aaliyah, a 32-year-old female weighing 68 kg and standing 1.75 meters tall, presents with severe anemia. Her current hematocrit is 21%. The physician wants to increase her hematocrit to 28%. Using the Nadler formula to estimate blood volume and considering that each unit of packed red blood cells (RBCs) has a volume of 300 mL and a hematocrit of 80%, how many units of packed RBCs are required to achieve the desired hematocrit? Assume no blood loss during the transfusion and that the Nadler formula for women is: \[Blood\,Volume (mL) = 0.3561 \times height(m)^3 + 0.03308 \times weight(kg) + 0.1833\]. Round your final answer to the nearest whole number.
Correct
To determine the required volume of red blood cells (RBCs) to increase a patient’s hematocrit, we first need to calculate the patient’s blood volume. We use the Nadler formula:
For males: \[Blood\,Volume (mL) = 0.3669 \times height(m)^3 + 0.03219 \times weight(kg) + 0.6041\]
For females: \[Blood\,Volume (mL) = 0.3561 \times height(m)^3 + 0.03308 \times weight(kg) + 0.1833\]Given that Aaliyah is female, her height is 1.75 meters, and her weight is 68 kg, her blood volume is:
\[Blood\,Volume = 0.3561 \times (1.75)^3 + 0.03308 \times 68 + 0.1833\]
\[Blood\,Volume = 0.3561 \times 5.359 + 2.249 + 0.1833\]
\[Blood\,Volume = 1.908 + 2.249 + 0.1833 = 4.3403\,L = 4340.3\,mL\]Next, we calculate the RBC volume in her current hematocrit:
\[Current\,RBC\,Volume = Blood\,Volume \times Hematocrit\]
\[Current\,RBC\,Volume = 4340.3\,mL \times 0.21 = 911.46\,mL\]We want to raise her hematocrit to 0.28. The required RBC volume is:
\[Required\,RBC\,Volume = Blood\,Volume \times Target\,Hematocrit\]
\[Required\,RBC\,Volume = 4340.3\,mL \times 0.28 = 1215.28\,mL\]The difference in RBC volume needed is:
\[RBC\,Volume\,Needed = Required\,RBC\,Volume – Current\,RBC\,Volume\]
\[RBC\,Volume\,Needed = 1215.28\,mL – 911.46\,mL = 303.82\,mL\]Each unit of packed RBCs has a volume of 300 mL and a hematocrit of 0.80. The actual RBC volume in each unit is:
\[RBC\,Volume\,per\,Unit = Unit\,Volume \times Hematocrit\]
\[RBC\,Volume\,per\,Unit = 300\,mL \times 0.80 = 240\,mL\]Finally, we calculate the number of units required:
\[Units\,Needed = \frac{RBC\,Volume\,Needed}{RBC\,Volume\,per\,Unit}\]
\[Units\,Needed = \frac{303.82\,mL}{240\,mL} = 1.265\]Since we cannot transfuse a fraction of a unit, we round up to the nearest whole number. Thus, 2 units are needed. This calculation involves understanding of blood volume estimation, hematocrit adjustments, and component characteristics, which are essential for SBB professionals in managing transfusion therapy.
Incorrect
To determine the required volume of red blood cells (RBCs) to increase a patient’s hematocrit, we first need to calculate the patient’s blood volume. We use the Nadler formula:
For males: \[Blood\,Volume (mL) = 0.3669 \times height(m)^3 + 0.03219 \times weight(kg) + 0.6041\]
For females: \[Blood\,Volume (mL) = 0.3561 \times height(m)^3 + 0.03308 \times weight(kg) + 0.1833\]Given that Aaliyah is female, her height is 1.75 meters, and her weight is 68 kg, her blood volume is:
\[Blood\,Volume = 0.3561 \times (1.75)^3 + 0.03308 \times 68 + 0.1833\]
\[Blood\,Volume = 0.3561 \times 5.359 + 2.249 + 0.1833\]
\[Blood\,Volume = 1.908 + 2.249 + 0.1833 = 4.3403\,L = 4340.3\,mL\]Next, we calculate the RBC volume in her current hematocrit:
\[Current\,RBC\,Volume = Blood\,Volume \times Hematocrit\]
\[Current\,RBC\,Volume = 4340.3\,mL \times 0.21 = 911.46\,mL\]We want to raise her hematocrit to 0.28. The required RBC volume is:
\[Required\,RBC\,Volume = Blood\,Volume \times Target\,Hematocrit\]
\[Required\,RBC\,Volume = 4340.3\,mL \times 0.28 = 1215.28\,mL\]The difference in RBC volume needed is:
\[RBC\,Volume\,Needed = Required\,RBC\,Volume – Current\,RBC\,Volume\]
\[RBC\,Volume\,Needed = 1215.28\,mL – 911.46\,mL = 303.82\,mL\]Each unit of packed RBCs has a volume of 300 mL and a hematocrit of 0.80. The actual RBC volume in each unit is:
\[RBC\,Volume\,per\,Unit = Unit\,Volume \times Hematocrit\]
\[RBC\,Volume\,per\,Unit = 300\,mL \times 0.80 = 240\,mL\]Finally, we calculate the number of units required:
\[Units\,Needed = \frac{RBC\,Volume\,Needed}{RBC\,Volume\,per\,Unit}\]
\[Units\,Needed = \frac{303.82\,mL}{240\,mL} = 1.265\]Since we cannot transfuse a fraction of a unit, we round up to the nearest whole number. Thus, 2 units are needed. This calculation involves understanding of blood volume estimation, hematocrit adjustments, and component characteristics, which are essential for SBB professionals in managing transfusion therapy.
-
Question 19 of 30
19. Question
A 62-year-old patient, Mr. Jian, with a history of coronary artery bypass grafting (CABG) five years ago and subsequent multiple packed red blood cell (PRBC) transfusions due to chronic anemia, presents to the blood bank for pre-operative compatibility testing before an elective hip replacement surgery. His initial antibody screen is negative. However, after repeat testing using a more sensitive method, a weak antibody is detected that reacts only at the antiglobulin (AHG) phase. Further testing reveals that the antibody reacts with only one of the three screening cells on the antibody screen panel. When the screening cells are enzyme-treated, the reactivity of the antibody significantly increases. Considering Mr. Jian’s transfusion history and the antibody’s reaction pattern, what is the most probable specificity of the antibody?
Correct
The scenario describes a situation where a patient with a history of multiple transfusions develops a new antibody that reacts at the AHG phase, but only with certain screening cells. Further investigation reveals that the antibody reacts more strongly with enzyme-treated red cells. This pattern of reactivity is characteristic of antibodies to the Rh blood group system, particularly anti-e, anti-c, or anti-Kidd antibodies. Enzyme treatment enhances the antigenicity of Rh and Kidd antigens, making them more reactive with their corresponding antibodies. The fact that the antibody reacts with enzyme-treated cells suggests that the antigen is sensitive to enzyme degradation. Lewis, M, and P1 antigens are destroyed by enzyme treatment, whereas Rh, Kidd, and I antigens are enhanced. Given the patient’s transfusion history, alloimmunization to a common red cell antigen is likely. The antibody’s enhanced reactivity with enzyme-treated cells and reactivity at the AHG phase points towards an Rh system antibody. Kidd antibodies are also enhanced by enzyme treatment but are less likely given the single cell reactivity described. Therefore, the most probable specificity is an Rh system antibody such as anti-e or anti-c.
Incorrect
The scenario describes a situation where a patient with a history of multiple transfusions develops a new antibody that reacts at the AHG phase, but only with certain screening cells. Further investigation reveals that the antibody reacts more strongly with enzyme-treated red cells. This pattern of reactivity is characteristic of antibodies to the Rh blood group system, particularly anti-e, anti-c, or anti-Kidd antibodies. Enzyme treatment enhances the antigenicity of Rh and Kidd antigens, making them more reactive with their corresponding antibodies. The fact that the antibody reacts with enzyme-treated cells suggests that the antigen is sensitive to enzyme degradation. Lewis, M, and P1 antigens are destroyed by enzyme treatment, whereas Rh, Kidd, and I antigens are enhanced. Given the patient’s transfusion history, alloimmunization to a common red cell antigen is likely. The antibody’s enhanced reactivity with enzyme-treated cells and reactivity at the AHG phase points towards an Rh system antibody. Kidd antibodies are also enhanced by enzyme treatment but are less likely given the single cell reactivity described. Therefore, the most probable specificity is an Rh system antibody such as anti-e or anti-c.
-
Question 20 of 30
20. Question
A 68-year-old patient, Maria Sanchez, presents to the blood bank with pre-operative orders for two units of packed red blood cells. Initial ABO/Rh typing reveals the following: Forward typing shows weak agglutination (1+) with both anti-A and anti-B reagents. A1 lectin also shows weak agglutination (1+). Reverse typing demonstrates strong agglutination (4+) with A1 cells and B cells. Anti-D typing is positive. The antibody screen is negative. Given these findings and adhering to AABB standards for discrepancy resolution, what is the MOST appropriate next step to resolve this ABO typing discrepancy and ensure the patient receives compatible blood?
Correct
The key to answering this question lies in understanding the nuances of ABO subgroup discrepancies, particularly in the context of *cis*-AB alleles and their serological expression. *Cis*-AB individuals inherit the A and B genes on the same chromosome, leading to a weakened expression of both A and B antigens. This weakened expression can cause discrepancies in forward and reverse typing. In this specific scenario, the patient demonstrates a weak reaction with anti-A and anti-B reagents, suggesting a quantitative reduction in antigen expression. The A1 lectin result is crucial; A1 lectin agglutinates A1 cells strongly but does not agglutinate A2 cells. The weak reaction with A1 lectin further points to a weakened A antigen expression. Reverse typing reveals a strong reaction with A1 cells and B cells, indicating the presence of anti-A1 and anti-B antibodies in the patient’s serum. This pattern is consistent with a *cis*-AB subgroup where both A and B antigens are weakly expressed, leading to the production of expected antibodies against the A1 antigen (due to the weakened A antigen expression) and the B antigen. The most appropriate course of action is to confirm the presence of the *cis*-AB allele through molecular testing. Molecular testing will definitively identify the genetic basis of the ABO subgroup and resolve the serological ambiguity. While adsorption/elution studies and family studies can provide valuable information, molecular testing offers the most precise and rapid resolution in this case.
Incorrect
The key to answering this question lies in understanding the nuances of ABO subgroup discrepancies, particularly in the context of *cis*-AB alleles and their serological expression. *Cis*-AB individuals inherit the A and B genes on the same chromosome, leading to a weakened expression of both A and B antigens. This weakened expression can cause discrepancies in forward and reverse typing. In this specific scenario, the patient demonstrates a weak reaction with anti-A and anti-B reagents, suggesting a quantitative reduction in antigen expression. The A1 lectin result is crucial; A1 lectin agglutinates A1 cells strongly but does not agglutinate A2 cells. The weak reaction with A1 lectin further points to a weakened A antigen expression. Reverse typing reveals a strong reaction with A1 cells and B cells, indicating the presence of anti-A1 and anti-B antibodies in the patient’s serum. This pattern is consistent with a *cis*-AB subgroup where both A and B antigens are weakly expressed, leading to the production of expected antibodies against the A1 antigen (due to the weakened A antigen expression) and the B antigen. The most appropriate course of action is to confirm the presence of the *cis*-AB allele through molecular testing. Molecular testing will definitively identify the genetic basis of the ABO subgroup and resolve the serological ambiguity. While adsorption/elution studies and family studies can provide valuable information, molecular testing offers the most precise and rapid resolution in this case.
-
Question 21 of 30
21. Question
A 55-year-old patient, Leticia Alvarez, with a history of thrombocytopenia secondary to chemotherapy, requires a platelet transfusion. Her physician orders a transfusion to increase her platelet count by 50,000/µL. Leticia’s body surface area (BSA) is determined to be 1.8 \(m^2\). The blood bank typically observes a platelet count increase of 30,000/µL per single platelet unit transfused in patients with similar conditions. According to AABB standards and considering the provided information, how many single platelet units should be transfused to achieve the desired platelet increment for Leticia?
Correct
To calculate the required number of platelets, we first need to determine the platelet increment needed. The desired increment is 50,000/µL, and the patient’s body surface area (BSA) is 1.8 \(m^2\). The formula to calculate the number of platelets needed is:
\[\text{Platelet Increment} = \text{Desired Increment} \times \text{BSA} \times \text{Platelet Count Increase per Platelet Unit} \]
We need to rearrange the formula to solve for the number of platelet units:
\[\text{Number of Platelet Units} = \frac{\text{Desired Increment} \times \text{BSA}}{\text{Platelet Count Increase per Platelet Unit}}\]
Given the desired increment is 50,000/µL, BSA is 1.8 \(m^2\), and the platelet count increase per platelet unit is 30,000/µL, we plug in the values:
\[\text{Number of Platelet Units} = \frac{50,000 \times 1.8}{30,000}\]
\[\text{Number of Platelet Units} = \frac{90,000}{30,000}\]
\[\text{Number of Platelet Units} = 3\]
Therefore, 3 platelet units are needed to achieve the desired increment of 50,000/µL for the patient.
Understanding platelet transfusion calculations is critical in blood banking to ensure appropriate dosing and patient safety. This involves considering the patient’s body surface area, the desired platelet increment, and the expected increase in platelet count per unit transfused. Accurate calculation prevents under- or over-transfusion, both of which can have adverse effects. Factors like splenomegaly, active bleeding, or platelet consumption can affect the post-transfusion platelet count, necessitating adjustments in the number of units transfused. Familiarity with these calculations and modifying factors is essential for SBB technologists to optimize transfusion therapy and manage patient outcomes effectively. Furthermore, adhering to established guidelines and protocols ensures consistency and safety in platelet transfusion practices.
Incorrect
To calculate the required number of platelets, we first need to determine the platelet increment needed. The desired increment is 50,000/µL, and the patient’s body surface area (BSA) is 1.8 \(m^2\). The formula to calculate the number of platelets needed is:
\[\text{Platelet Increment} = \text{Desired Increment} \times \text{BSA} \times \text{Platelet Count Increase per Platelet Unit} \]
We need to rearrange the formula to solve for the number of platelet units:
\[\text{Number of Platelet Units} = \frac{\text{Desired Increment} \times \text{BSA}}{\text{Platelet Count Increase per Platelet Unit}}\]
Given the desired increment is 50,000/µL, BSA is 1.8 \(m^2\), and the platelet count increase per platelet unit is 30,000/µL, we plug in the values:
\[\text{Number of Platelet Units} = \frac{50,000 \times 1.8}{30,000}\]
\[\text{Number of Platelet Units} = \frac{90,000}{30,000}\]
\[\text{Number of Platelet Units} = 3\]
Therefore, 3 platelet units are needed to achieve the desired increment of 50,000/µL for the patient.
Understanding platelet transfusion calculations is critical in blood banking to ensure appropriate dosing and patient safety. This involves considering the patient’s body surface area, the desired platelet increment, and the expected increase in platelet count per unit transfused. Accurate calculation prevents under- or over-transfusion, both of which can have adverse effects. Factors like splenomegaly, active bleeding, or platelet consumption can affect the post-transfusion platelet count, necessitating adjustments in the number of units transfused. Familiarity with these calculations and modifying factors is essential for SBB technologists to optimize transfusion therapy and manage patient outcomes effectively. Furthermore, adhering to established guidelines and protocols ensures consistency and safety in platelet transfusion practices.
-
Question 22 of 30
22. Question
A 28-year-old pregnant woman, Maria, at 28 weeks gestation, is undergoing routine prenatal testing. Her initial ABO/Rh typing results indicate that she is A positive. However, the weak D test is positive. Further investigation at a reference laboratory reveals that Maria has a partial D phenotype, specifically DVI. The obstetrician consults with the blood bank medical director regarding the appropriate course of action concerning RhIg administration. The reference lab reports that individuals with DVI phenotype are capable of producing anti-D alloantibodies if exposed to complete D antigen. Considering this information and current AABB standards, what is the MOST appropriate course of action regarding RhIg administration for Maria?
Correct
The correct answer involves understanding the complexities of RhIg administration, particularly in cases where the mother might have a partial D antigen. RhIg works by preventing alloimmunization to the D antigen. However, if a woman has a partial D phenotype, she can produce alloantibodies against the parts of the D antigen that she lacks. Standard RhIg will not protect against this alloimmunization because it only targets the complete D antigen. The key consideration is whether the partial D phenotype is capable of eliciting an anti-D response. If the patient is considered D-positive by routine testing but is actually a partial D and makes antibody to the “missing” D epitopes, RhIg would not prevent this alloimmunization. The decision to administer RhIg in such cases depends on whether the partial D type is likely to make anti-D. If the partial D type is unlikely to make anti-D, RhIg may be administered. If there is doubt, further testing and consultation with a reference laboratory are essential. The reference lab can determine if the patient is at risk of making anti-D. If the patient is at risk, RhIg would not be appropriate, and the patient should be managed as Rh-negative.
Incorrect
The correct answer involves understanding the complexities of RhIg administration, particularly in cases where the mother might have a partial D antigen. RhIg works by preventing alloimmunization to the D antigen. However, if a woman has a partial D phenotype, she can produce alloantibodies against the parts of the D antigen that she lacks. Standard RhIg will not protect against this alloimmunization because it only targets the complete D antigen. The key consideration is whether the partial D phenotype is capable of eliciting an anti-D response. If the patient is considered D-positive by routine testing but is actually a partial D and makes antibody to the “missing” D epitopes, RhIg would not prevent this alloimmunization. The decision to administer RhIg in such cases depends on whether the partial D type is likely to make anti-D. If the partial D type is unlikely to make anti-D, RhIg may be administered. If there is doubt, further testing and consultation with a reference laboratory are essential. The reference lab can determine if the patient is at risk of making anti-D. If the patient is at risk, RhIg would not be appropriate, and the patient should be managed as Rh-negative.
-
Question 23 of 30
23. Question
A 68-year-old patient, Ms. Eleanor Vance, with a history of multiple red blood cell transfusions due to chronic anemia, presents to the blood bank with a persistently positive antibody screen. Initial antibody identification using a standard 11-cell panel reveals the presence of anti-E. Despite adsorption with E-positive cells, the antibody screen remains reactive, albeit at a lower titer. Further investigations, including enzyme treatment and extended panels, fail to identify any additional common alloantibodies. The blood bank technologist suspects the presence of an antibody to a low-incidence antigen masked by the anti-E. Following AABB guidelines for complex antibody workups, what is the MOST appropriate next step to resolve this complex serological picture and ensure compatible blood is available for Ms. Vance?
Correct
The scenario describes a complex case involving a patient with a history of multiple transfusions and a persistently reactive antibody screen despite extensive antibody identification procedures. The key to resolving this case lies in understanding the limitations of standard serological techniques and recognizing the potential for underlying genetic variations that can influence antigen expression. Adsorption studies are crucial for removing interfering antibodies and revealing underlying specificities. In this case, performing adsorption studies using R1R1 and R2R2 cells can help determine if an antibody to a low-incidence antigen is present, masked by the anti-E. Once the anti-E is removed, if reactivity persists with certain panel cells, adsorption with cells expressing common antigens may be needed to further isolate the underlying antibody. Molecular testing could confirm the presence of rare alleles. This approach aligns with best practices for complex antibody identification as recommended by AABB and other regulatory bodies, focusing on resolving discrepancies and ensuring patient safety in transfusion.
Incorrect
The scenario describes a complex case involving a patient with a history of multiple transfusions and a persistently reactive antibody screen despite extensive antibody identification procedures. The key to resolving this case lies in understanding the limitations of standard serological techniques and recognizing the potential for underlying genetic variations that can influence antigen expression. Adsorption studies are crucial for removing interfering antibodies and revealing underlying specificities. In this case, performing adsorption studies using R1R1 and R2R2 cells can help determine if an antibody to a low-incidence antigen is present, masked by the anti-E. Once the anti-E is removed, if reactivity persists with certain panel cells, adsorption with cells expressing common antigens may be needed to further isolate the underlying antibody. Molecular testing could confirm the presence of rare alleles. This approach aligns with best practices for complex antibody identification as recommended by AABB and other regulatory bodies, focusing on resolving discrepancies and ensuring patient safety in transfusion.
-
Question 24 of 30
24. Question
Dr. Anya Sharma, a hematologist, requests a platelet transfusion for her patient, Mr. Jian Li, who weighs 80 kg and is 180 cm tall. Mr. Li has a stable platelet count of 15,000/µL and requires a transfusion to increase his platelet count to 20,000/µL before a scheduled bone marrow biopsy. The blood bank uses apheresis platelet units with an average platelet count of \(3.0 \times 10^{11}\) platelets per unit. Assuming a platelet recovery rate of 60% in Mr. Li due to his splenomegaly, calculate the number of apheresis platelet units required to achieve the desired platelet increment. What is the minimum number of whole apheresis platelet units needed for transfusion?
Correct
To calculate the required number of platelets, we first need to determine the desired platelet increment. The desired increment is 5,000/µL. Next, we estimate the patient’s body surface area (BSA). Given the patient’s weight (80 kg) and height (180 cm), we can use the Mosteller formula: \(BSA = \sqrt{\frac{height(cm) \times weight(kg)}{3600}}\).
\[BSA = \sqrt{\frac{180 \times 80}{3600}} = \sqrt{\frac{14400}{3600}} = \sqrt{4} = 2 \, m^2\]
Now, we calculate the number of platelets needed using the formula:
\[\text{Number of platelets needed} = \frac{\text{Desired increment} \times BSA(m^2)}{\text{Platelet count per unit} \times \text{Platelet recovery}}\]
Given a platelet count per apheresis unit of \(3.0 \times 10^{11}\) and a platelet recovery of 60% (0.6), the formula becomes:
\[\text{Number of platelets needed} = \frac{5000/\mu L \times 10^6 \mu L/L \times 2 \, m^2}{3.0 \times 10^{11} \times 0.6} = \frac{10 \times 10^9}{1.8 \times 10^{11}} = \frac{10^{10}}{1.8 \times 10^{11}} = \frac{1}{18} \times 10 = 0.0556 \times 10 = 5.56 \text{ units}\]
Since we cannot transfuse a fraction of a unit, we round up to the nearest whole number. Therefore, 6 apheresis platelet units are required. Understanding BSA calculation and platelet transfusion formulas are crucial in determining appropriate platelet dosage, especially in patients with complex medical conditions. Platelet recovery rates can vary significantly based on the patient’s clinical condition, such as splenomegaly, DIC, or alloimmunization.Incorrect
To calculate the required number of platelets, we first need to determine the desired platelet increment. The desired increment is 5,000/µL. Next, we estimate the patient’s body surface area (BSA). Given the patient’s weight (80 kg) and height (180 cm), we can use the Mosteller formula: \(BSA = \sqrt{\frac{height(cm) \times weight(kg)}{3600}}\).
\[BSA = \sqrt{\frac{180 \times 80}{3600}} = \sqrt{\frac{14400}{3600}} = \sqrt{4} = 2 \, m^2\]
Now, we calculate the number of platelets needed using the formula:
\[\text{Number of platelets needed} = \frac{\text{Desired increment} \times BSA(m^2)}{\text{Platelet count per unit} \times \text{Platelet recovery}}\]
Given a platelet count per apheresis unit of \(3.0 \times 10^{11}\) and a platelet recovery of 60% (0.6), the formula becomes:
\[\text{Number of platelets needed} = \frac{5000/\mu L \times 10^6 \mu L/L \times 2 \, m^2}{3.0 \times 10^{11} \times 0.6} = \frac{10 \times 10^9}{1.8 \times 10^{11}} = \frac{10^{10}}{1.8 \times 10^{11}} = \frac{1}{18} \times 10 = 0.0556 \times 10 = 5.56 \text{ units}\]
Since we cannot transfuse a fraction of a unit, we round up to the nearest whole number. Therefore, 6 apheresis platelet units are required. Understanding BSA calculation and platelet transfusion formulas are crucial in determining appropriate platelet dosage, especially in patients with complex medical conditions. Platelet recovery rates can vary significantly based on the patient’s clinical condition, such as splenomegaly, DIC, or alloimmunization. -
Question 25 of 30
25. Question
A 28-year-old primigravida, Ms. Anya Sharma, is Rh-negative and has a negative antibody screen at 28 weeks gestation. She delivers a healthy baby boy at term. Postpartum, the infant’s blood type is determined to be A positive, weak D positive, with a negative direct antiglobulin test (DAT). Ms. Sharma’s postpartum antibody screen remains negative. According to current AABB standards and accepted clinical practice, which of the following is the MOST appropriate course of action regarding Rh immune globulin (RhIg) administration for Ms. Sharma? Consider all aspects of the case and select the option that best reflects established protocol.
Correct
The key to this scenario lies in understanding the nuances of RhIg administration guidelines, particularly in cases involving weak D phenotypes. While RhIg is routinely administered to Rh-negative mothers to prevent alloimmunization, the situation becomes more complex when the infant presents with a weak D phenotype. According to AABB standards and accepted clinical practice, RhIg administration is indicated when the infant is either Rh-positive or weak D positive, *and* the mother is Rh-negative *and* has not developed anti-D. The purpose is to prevent the mother from forming anti-D if fetal-maternal hemorrhage has occurred. If the mother has already formed anti-D, RhIg will not be effective and is not indicated. The direct antiglobulin test (DAT) result on the infant is important in evaluating HDFN but does not dictate the RhIg decision *for the mother*. The critical factor is the *mother’s* Rh status and antibody screen results. Since the mother is Rh-negative and has a negative antibody screen, she is a candidate for RhIg. The infant’s weak D status confirms the presence of D antigen, albeit weakly expressed. Therefore, RhIg should be administered to the mother. The dosage is based on the estimated volume of fetal-maternal hemorrhage, typically assessed via a Kleihauer-Betke test or a similar method, but a standard dose is often given prophylactically. The weak D testing of the infant should have been performed using an indirect antiglobulin test (IAT).
Incorrect
The key to this scenario lies in understanding the nuances of RhIg administration guidelines, particularly in cases involving weak D phenotypes. While RhIg is routinely administered to Rh-negative mothers to prevent alloimmunization, the situation becomes more complex when the infant presents with a weak D phenotype. According to AABB standards and accepted clinical practice, RhIg administration is indicated when the infant is either Rh-positive or weak D positive, *and* the mother is Rh-negative *and* has not developed anti-D. The purpose is to prevent the mother from forming anti-D if fetal-maternal hemorrhage has occurred. If the mother has already formed anti-D, RhIg will not be effective and is not indicated. The direct antiglobulin test (DAT) result on the infant is important in evaluating HDFN but does not dictate the RhIg decision *for the mother*. The critical factor is the *mother’s* Rh status and antibody screen results. Since the mother is Rh-negative and has a negative antibody screen, she is a candidate for RhIg. The infant’s weak D status confirms the presence of D antigen, albeit weakly expressed. Therefore, RhIg should be administered to the mother. The dosage is based on the estimated volume of fetal-maternal hemorrhage, typically assessed via a Kleihauer-Betke test or a similar method, but a standard dose is often given prophylactically. The weak D testing of the infant should have been performed using an indirect antiglobulin test (IAT).
-
Question 26 of 30
26. Question
A 68-year-old patient, Maria Rodriguez, requires a pre-transfusion compatibility test. Her red cells agglutinate strongly (4+) with anti-A and anti-A,B, but show no agglutination with anti-A1 lectin. The autocontrol is negative. The reverse grouping demonstrates a strong (4+) reaction with B cells and a negative reaction with A1 cells. The technologist suspects an ABO subgroup discrepancy. Given these findings, which of the following is the MOST appropriate next step to resolve this discrepancy and ensure appropriate blood product selection according to AABB standards and FDA regulations related to blood banking practices? The blood bank adheres strictly to SOPs derived from the AABB Technical Manual and follows guidelines for ABO discrepancy resolution as outlined in relevant regulatory documents.
Correct
The key to this question lies in understanding the complexities of ABO subgroups, particularly the A subgroups and their reactions with different anti-A reagents. A2 cells react strongly with anti-A but may show weaker or negative reactions with anti-A1 lectin (Dolichos biflorus). Anti-H lectin reacts more strongly with O and A2 cells than with A1 cells because A1 cells have more of their H antigen converted to A antigen. The patient’s cells reacting with anti-A and not with anti-A1 lectin suggests an A2 subgroup. The anti-A,B reactivity confirms the presence of an A antigen. The negative autocontrol rules out autoagglutination. The most appropriate course of action is to perform an anti-A1 lectin absorption study. This is done by incubating the patient’s serum with A1 cells to absorb any anti-A1 present. If the patient is A2 and has an anti-A1, this absorption will remove the anti-A1 reactivity. After absorption, the serum is tested against A1 and A2 cells. If the reactivity against A1 cells is removed or significantly reduced while the reactivity against A2 cells remains unchanged, it confirms the presence of anti-A1 in an A2 individual. This is a common and well-documented phenomenon. Performing a family study, while potentially informative, is not the most immediate or necessary step in this situation. Reporting the patient as group A without further investigation could lead to transfusion errors if the patient receives A1 blood. Performing an antibody screen is not the most direct approach to resolving this specific ABO discrepancy.
Incorrect
The key to this question lies in understanding the complexities of ABO subgroups, particularly the A subgroups and their reactions with different anti-A reagents. A2 cells react strongly with anti-A but may show weaker or negative reactions with anti-A1 lectin (Dolichos biflorus). Anti-H lectin reacts more strongly with O and A2 cells than with A1 cells because A1 cells have more of their H antigen converted to A antigen. The patient’s cells reacting with anti-A and not with anti-A1 lectin suggests an A2 subgroup. The anti-A,B reactivity confirms the presence of an A antigen. The negative autocontrol rules out autoagglutination. The most appropriate course of action is to perform an anti-A1 lectin absorption study. This is done by incubating the patient’s serum with A1 cells to absorb any anti-A1 present. If the patient is A2 and has an anti-A1, this absorption will remove the anti-A1 reactivity. After absorption, the serum is tested against A1 and A2 cells. If the reactivity against A1 cells is removed or significantly reduced while the reactivity against A2 cells remains unchanged, it confirms the presence of anti-A1 in an A2 individual. This is a common and well-documented phenomenon. Performing a family study, while potentially informative, is not the most immediate or necessary step in this situation. Reporting the patient as group A without further investigation could lead to transfusion errors if the patient receives A1 blood. Performing an antibody screen is not the most direct approach to resolving this specific ABO discrepancy.
-
Question 27 of 30
27. Question
A 75 kg patient with severe anemia has a current hemoglobin level of 7 g/dL. The physician wants to increase the patient’s hemoglobin level to 10 g/dL using packed red blood cells (PRBCs). Assuming the patient has a normal blood volume for their weight and that each unit of PRBCs will contribute approximately 50g of hemoglobin to the patient’s total hemoglobin mass, how many units of PRBCs should be transfused to achieve the desired hemoglobin level?
Correct
To determine the required volume of packed red blood cells (PRBCs), we need to calculate the difference between the desired hemoglobin level and the patient’s current hemoglobin level, and then use a formula that accounts for the expected increase in hemoglobin per unit of PRBCs transfused and the patient’s blood volume. The standard formula used is:
\[ \text{Volume of PRBCs (mL)} = \frac{\text{Desired Hb increase (g/dL)} \times \text{Patient Blood Volume (mL)}}{\text{Hb increase per unit (g/dL)}} \]
First, calculate the desired Hb increase:
Desired Hb increase = Desired Hb – Current Hb = 10 g/dL – 7 g/dL = 3 g/dLNext, calculate the patient’s blood volume. We can estimate blood volume using the following formula:
Blood Volume (mL) = Patient Weight (kg) x Blood Volume Factor (mL/kg)
Assuming a blood volume factor of 70 mL/kg for adults:
Blood Volume = 75 kg x 70 mL/kg = 5250 mLNow, we need to know the expected hemoglobin increase per unit of PRBCs transfused. Typically, one unit of PRBCs (approximately 300 mL) will increase the hemoglobin level by about 1 g/dL in an average adult.
Using the formula:
\[ \text{Volume of PRBCs (mL)} = \frac{3 \text{ g/dL} \times 5250 \text{ mL}}{1 \text{ g/dL}} = 15750 \text{ mL} \]Now, determine the number of units needed. If each unit is approximately 300 mL:
Number of units = Total volume needed / Volume per unit = 15750 mL / 300 mL/unit = 52.5 unitsSince we can’t transfuse half a unit, we round up to 53 units.
However, this is an extremely high number of units, indicating that the 1 g/dL increase per unit assumption is not valid in this scenario. The question is asking about a targeted increase to 10 g/dL. The calculation should be based on the *total* volume of PRBCs needed to achieve this level.
The more accurate approach considers the total red cell mass required. If we assume each unit of PRBCs contains approximately 50g of hemoglobin (a reasonable approximation):
Total Hb needed = Desired Hb level (g/dL) x Blood volume (dL) = 10 g/dL * 52.5 dL = 525 g
Current total Hb = Current Hb level (g/dL) x Blood volume (dL) = 7 g/dL * 52.5 dL = 367.5 g
Hb deficit = Total Hb needed – Current total Hb = 525 g – 367.5 g = 157.5 g
Units of PRBCs = Hb deficit / Hb per unit = 157.5 g / 50 g/unit = 3.15 units
Round up to 4 units.
Incorrect
To determine the required volume of packed red blood cells (PRBCs), we need to calculate the difference between the desired hemoglobin level and the patient’s current hemoglobin level, and then use a formula that accounts for the expected increase in hemoglobin per unit of PRBCs transfused and the patient’s blood volume. The standard formula used is:
\[ \text{Volume of PRBCs (mL)} = \frac{\text{Desired Hb increase (g/dL)} \times \text{Patient Blood Volume (mL)}}{\text{Hb increase per unit (g/dL)}} \]
First, calculate the desired Hb increase:
Desired Hb increase = Desired Hb – Current Hb = 10 g/dL – 7 g/dL = 3 g/dLNext, calculate the patient’s blood volume. We can estimate blood volume using the following formula:
Blood Volume (mL) = Patient Weight (kg) x Blood Volume Factor (mL/kg)
Assuming a blood volume factor of 70 mL/kg for adults:
Blood Volume = 75 kg x 70 mL/kg = 5250 mLNow, we need to know the expected hemoglobin increase per unit of PRBCs transfused. Typically, one unit of PRBCs (approximately 300 mL) will increase the hemoglobin level by about 1 g/dL in an average adult.
Using the formula:
\[ \text{Volume of PRBCs (mL)} = \frac{3 \text{ g/dL} \times 5250 \text{ mL}}{1 \text{ g/dL}} = 15750 \text{ mL} \]Now, determine the number of units needed. If each unit is approximately 300 mL:
Number of units = Total volume needed / Volume per unit = 15750 mL / 300 mL/unit = 52.5 unitsSince we can’t transfuse half a unit, we round up to 53 units.
However, this is an extremely high number of units, indicating that the 1 g/dL increase per unit assumption is not valid in this scenario. The question is asking about a targeted increase to 10 g/dL. The calculation should be based on the *total* volume of PRBCs needed to achieve this level.
The more accurate approach considers the total red cell mass required. If we assume each unit of PRBCs contains approximately 50g of hemoglobin (a reasonable approximation):
Total Hb needed = Desired Hb level (g/dL) x Blood volume (dL) = 10 g/dL * 52.5 dL = 525 g
Current total Hb = Current Hb level (g/dL) x Blood volume (dL) = 7 g/dL * 52.5 dL = 367.5 g
Hb deficit = Total Hb needed – Current total Hb = 525 g – 367.5 g = 157.5 g
Units of PRBCs = Hb deficit / Hb per unit = 157.5 g / 50 g/unit = 3.15 units
Round up to 4 units.
-
Question 28 of 30
28. Question
A 68-year-old patient, Ms. Eleanor Vance, with a history of multiple red blood cell transfusions due to chronic anemia, presents to the emergency department with acute blood loss following a motor vehicle accident. Her pre-transfusion sample demonstrates a positive antibody screen. Further investigation reveals the presence of a warm autoantibody. The blood bank scientist is unable to definitively exclude the presence of an underlying alloantibody using routine antibody identification techniques. The patient’s physician urgently requests two units of packed red blood cells. Autologous units are unavailable. According to AABB standards and best practices for complex antibody problems, what is the MOST appropriate next step to ensure provision of compatible blood for Ms. Vance?
Correct
The scenario describes a complex case involving a patient with a history of multiple transfusions and a warm autoantibody, complicated by the presence of an alloantibody. The key is to select the most appropriate course of action to ensure compatible blood is provided for the patient.
Warming the patient’s sample to 37°C will enhance the reactivity of the warm autoantibody, potentially masking any underlying alloantibodies. While this might seem counterintuitive, the goal is to saturate the autoantibody reactivity so that any remaining reactivity is more likely to be due to alloantibodies. Further adsorption studies are then performed to remove the autoantibody, revealing any underlying alloantibodies. Once the autoantibody is removed, the remaining serum can be tested against an antibody panel to identify the alloantibody specificity. Only after the alloantibody is identified can antigen-negative units be selected for crossmatching. Direct selection of random units for crossmatching is inappropriate because the presence of the alloantibody means that not all units will be compatible. Performing an elution study is also not the most appropriate initial step, as it primarily identifies antibodies coating the patient’s red cells, rather than free antibodies in the serum that would cause immediate transfusion reactions. Finally, transfusing the patient with autologous units is ideal if available, but the question states these are not an option. Therefore, the best course of action is to perform warm autoabsorption studies.
Incorrect
The scenario describes a complex case involving a patient with a history of multiple transfusions and a warm autoantibody, complicated by the presence of an alloantibody. The key is to select the most appropriate course of action to ensure compatible blood is provided for the patient.
Warming the patient’s sample to 37°C will enhance the reactivity of the warm autoantibody, potentially masking any underlying alloantibodies. While this might seem counterintuitive, the goal is to saturate the autoantibody reactivity so that any remaining reactivity is more likely to be due to alloantibodies. Further adsorption studies are then performed to remove the autoantibody, revealing any underlying alloantibodies. Once the autoantibody is removed, the remaining serum can be tested against an antibody panel to identify the alloantibody specificity. Only after the alloantibody is identified can antigen-negative units be selected for crossmatching. Direct selection of random units for crossmatching is inappropriate because the presence of the alloantibody means that not all units will be compatible. Performing an elution study is also not the most appropriate initial step, as it primarily identifies antibodies coating the patient’s red cells, rather than free antibodies in the serum that would cause immediate transfusion reactions. Finally, transfusing the patient with autologous units is ideal if available, but the question states these are not an option. Therefore, the best course of action is to perform warm autoabsorption studies.
-
Question 29 of 30
29. Question
A 65-year-old patient, Ms. Eleanor Vance, with a history of multiple red blood cell transfusions due to chronic anemia, presents to the blood bank for pre-transfusion testing. Her ABO type is A positive. Antibody screening is positive at both immediate spin (IS) and the anti-human globulin (AHG) phases. A 11-cell antibody panel is performed, showing reactions with cells 1, 2, 4, 6, 8, and 10 at AHG, with varying strengths (1+ to 3+). Cells 1 and 6 also show weak reactions (1+) at IS. Enzyme treatment of the panel cells enhances the reactions with cells 1, 2, 6, and 10, while abolishing the reactions with cells 4 and 8. Auto-control is negative. Given these results, which of the following antibody combinations is the MOST likely explanation for Ms. Vance’s antibody profile, considering the effects of enzyme treatment and reaction phases?
Correct
The scenario presents a complex antibody identification case involving multiple specificities and reactivity patterns. The initial panel reveals reactions at both immediate spin (IS) and anti-human globulin (AHG) phases, with varying strengths. Enzyme treatment enhances some reactions while abolishing others, suggesting the presence of antibodies to enzyme-sensitive and enzyme-resistant antigens. Cold reactive antibodies must also be considered. The patient’s history of multiple transfusions increases the likelihood of alloantibody formation. The key to resolving this case lies in carefully analyzing the reaction patterns, considering the effects of enzyme treatment, and utilizing techniques like antibody neutralization or adsorption to isolate individual specificities.
The most likely combination is anti-E, anti-K, and anti-Lea. Anti-E commonly reacts at AHG phase and can be enhanced by enzyme treatment. Anti-K is typically an AHG reactive antibody and is not affected by enzyme treatment. Anti-Lea is typically a cold reactive antibody but can react at AHG and is destroyed by enzyme treatment. The combination of these antibodies accounts for the observed reactivity patterns and the effects of enzyme treatment.
Incorrect
The scenario presents a complex antibody identification case involving multiple specificities and reactivity patterns. The initial panel reveals reactions at both immediate spin (IS) and anti-human globulin (AHG) phases, with varying strengths. Enzyme treatment enhances some reactions while abolishing others, suggesting the presence of antibodies to enzyme-sensitive and enzyme-resistant antigens. Cold reactive antibodies must also be considered. The patient’s history of multiple transfusions increases the likelihood of alloantibody formation. The key to resolving this case lies in carefully analyzing the reaction patterns, considering the effects of enzyme treatment, and utilizing techniques like antibody neutralization or adsorption to isolate individual specificities.
The most likely combination is anti-E, anti-K, and anti-Lea. Anti-E commonly reacts at AHG phase and can be enhanced by enzyme treatment. Anti-K is typically an AHG reactive antibody and is not affected by enzyme treatment. Anti-Lea is typically a cold reactive antibody but can react at AHG and is destroyed by enzyme treatment. The combination of these antibodies accounts for the observed reactivity patterns and the effects of enzyme treatment.
-
Question 30 of 30
30. Question
Dr. Anya Sharma, a hematologist at City General Hospital, is preparing a transfusion order for a 55-year-old patient, Mr. Jian Li, who weighs 70 kg and has a body surface area (BSA) of 1.8 \(m^2\). Mr. Li is severely thrombocytopenic due to chemotherapy and requires a platelet transfusion to increase his platelet count by 50,000/µL. The blood bank uses apheresis platelets, with each unit containing approximately \(3 \times 10^{11}\) platelets. Assuming the patient’s platelet recovery is expected to be 60% due to splenomegaly and the desired platelet support interval is 24 hours, how many apheresis platelet units should Dr. Sharma order to achieve the target platelet increment? This scenario requires careful consideration of platelet increment calculations, BSA adjustments, and the impact of platelet recovery on transfusion outcomes.
Correct
To calculate the required number of platelets, we must first determine the platelet increment needed and then adjust for factors like platelet support interval and platelet recovery.
1. **Calculate the required platelet increment:** The patient needs an increment of 50,000/µL. Given the patient’s body surface area (BSA) of 1.8 \(m^2\), we calculate the total number of platelets needed:
\[
\text{Platelet Increment Needed} = \text{Desired Increment} \times \text{BSA} = 50,000/\mu L \times 1.8 \, m^2 = 90,000
\]2. **Adjust for platelet support interval:** The desired platelet support interval is 24 hours.
3. **Adjust for platelet recovery:** The expected platelet recovery is 60%. This means only 60% of the transfused platelets will be effectively circulating. Therefore, we need to compensate for this loss:
\[
\text{Adjusted Platelet Count} = \frac{\text{Platelet Increment Needed}}{\text{Platelet Recovery}} = \frac{90,000}{0.60} = 150,000
\]4. **Calculate the number of platelet units needed:** Each apheresis platelet unit contains \(3 \times 10^{11}\) platelets. To find out how many units are required, we divide the adjusted platelet count by the number of platelets per unit:
\[
\text{Number of Platelet Units} = \frac{\text{Adjusted Platelet Count}}{3 \times 10^{11}} = \frac{150,000 \times 10^6}{3 \times 10^{11}} = \frac{1.5 \times 10^{11}}{3 \times 10^{11}} = 0.5
\]Since we can’t transfuse half a unit, we round up to ensure the patient receives adequate platelets. However, since the calculation is based on per microliter and per unit measurements, we need to consider the units of the increment calculation.
The platelet increment calculation yields \(90,000 \times 10^6\) platelets needed, and after adjusting for recovery, we get \(150,000 \times 10^6\) platelets. This value represents the total number of platelets needed based on the BSA. Now, we determine how many platelet units are required to meet this need, considering each unit contains \(3 \times 10^{11}\) platelets.
\[
\text{Number of Platelet Units} = \frac{150,000 \times 10^6}{3 \times 10^{11}} = 0.5
\]This means we need 0.5 units to achieve the desired increment after accounting for platelet recovery. Since the question asks for the number of apheresis platelets to order, and we cannot order half a unit, we must order a full unit to meet the requirements, ensuring adequate platelet count post-transfusion.
Therefore, one apheresis platelet unit is needed.
This question tests the candidate’s ability to apply theoretical knowledge to a practical transfusion scenario, emphasizing the importance of accurate calculations and adjustments for factors affecting platelet transfusion efficacy. The scenario requires understanding of platelet increment calculations, BSA adjustments, and the impact of platelet recovery on transfusion outcomes. It assesses the ability to integrate these concepts to determine the appropriate number of platelet units to order.
Incorrect
To calculate the required number of platelets, we must first determine the platelet increment needed and then adjust for factors like platelet support interval and platelet recovery.
1. **Calculate the required platelet increment:** The patient needs an increment of 50,000/µL. Given the patient’s body surface area (BSA) of 1.8 \(m^2\), we calculate the total number of platelets needed:
\[
\text{Platelet Increment Needed} = \text{Desired Increment} \times \text{BSA} = 50,000/\mu L \times 1.8 \, m^2 = 90,000
\]2. **Adjust for platelet support interval:** The desired platelet support interval is 24 hours.
3. **Adjust for platelet recovery:** The expected platelet recovery is 60%. This means only 60% of the transfused platelets will be effectively circulating. Therefore, we need to compensate for this loss:
\[
\text{Adjusted Platelet Count} = \frac{\text{Platelet Increment Needed}}{\text{Platelet Recovery}} = \frac{90,000}{0.60} = 150,000
\]4. **Calculate the number of platelet units needed:** Each apheresis platelet unit contains \(3 \times 10^{11}\) platelets. To find out how many units are required, we divide the adjusted platelet count by the number of platelets per unit:
\[
\text{Number of Platelet Units} = \frac{\text{Adjusted Platelet Count}}{3 \times 10^{11}} = \frac{150,000 \times 10^6}{3 \times 10^{11}} = \frac{1.5 \times 10^{11}}{3 \times 10^{11}} = 0.5
\]Since we can’t transfuse half a unit, we round up to ensure the patient receives adequate platelets. However, since the calculation is based on per microliter and per unit measurements, we need to consider the units of the increment calculation.
The platelet increment calculation yields \(90,000 \times 10^6\) platelets needed, and after adjusting for recovery, we get \(150,000 \times 10^6\) platelets. This value represents the total number of platelets needed based on the BSA. Now, we determine how many platelet units are required to meet this need, considering each unit contains \(3 \times 10^{11}\) platelets.
\[
\text{Number of Platelet Units} = \frac{150,000 \times 10^6}{3 \times 10^{11}} = 0.5
\]This means we need 0.5 units to achieve the desired increment after accounting for platelet recovery. Since the question asks for the number of apheresis platelets to order, and we cannot order half a unit, we must order a full unit to meet the requirements, ensuring adequate platelet count post-transfusion.
Therefore, one apheresis platelet unit is needed.
This question tests the candidate’s ability to apply theoretical knowledge to a practical transfusion scenario, emphasizing the importance of accurate calculations and adjustments for factors affecting platelet transfusion efficacy. The scenario requires understanding of platelet increment calculations, BSA adjustments, and the impact of platelet recovery on transfusion outcomes. It assesses the ability to integrate these concepts to determine the appropriate number of platelet units to order.